National NORCET 6

You might also like

Download as pdf or txt
Download as pdf or txt
You are on page 1of 111

National NORCET Test 6

ANATOMY & PHYSIOLOGY


1. Each cerebral hemisphere is divided into how many lobes?
a. 2
b. 3
c. 4
d. 5

Ans. c. 4

Rationale:
• The cerebrum is the largest part of the brain. It is divided into two hemispheres which
are each further sub-divided into four lobes. These lobes – frontal, temporal, parietal
and occipital lobes – are named after the cranial bones that lie over them.
• Each lobe has anatomical and functional differences.
• The frontal lobe influences personality, judgment, abstract reasoning, social behavior,
language expression and voluntary movement.
• The temporal lobe controls hearing, understanding of language, learning, storage and
recall of memories.
• The parietal lobe interprets and integrates sensation, inducing pain, temperature and
touch. It also interprets size, shape, distance, vibration and texture.
• The occipital lobe mainly interprets visual stimuli from the eyes.

2. A male client with a gunshot wound requires an emergency blood transfusion.


His blood type is AB negative. Which blood type would be the safest for him
to receive?
a. A Rh-positive
b. A Rh-negative
c. AB Rh-positive
d. O Rh-positive

Ans. b. A Rh-negative
2 National NORCET Test 6

Rationale:
• Human blood can sometimes contain an inherited D antigen. Persons with the D
antigen have Rh-positive blood type; those lacking the antigen have Rh-negative
blood. It’s important that a person with Rh-negative blood receives Rh-negative blood.
• If Rh-positive blood is administered to an Rh-negative person, the recipient develops
anti-Rh agglutinins, and subsequent transfusions with Rh-positive blood may cause
serious reactions with clumping and hemolysis of red blood cells.
• O Rh-positive blood type is still not compatible because it is Rh-positive.

3. Creatinine is a metabolic waste product excreted in urine and derived from:


a. Liver
b. Muscle
c. Bone
d. Skin

Ans. b. Muscle

Rationale:
• Creatinine is a break-down product of creatine phosphate in muscle and is usually
produced at a constant rate by the body (depending on muscle mass).
• Creatinine is chiefly filtered from the blood by the kidneys, although a small amount is
actively secreted by the kidneys into the urine.
• There is little or no tubular reabsorption of creatinine. If the filtering of the kidney is
deficient, blood creatinine levels rise. Therefore, creatinine levels in blood and urine
may be used to calculate creatinine clearance, which reflects the glomerular filtration
rate (GFR). This is clinically important because it is a measurement of renal function.

4. The wandering phagocytes found in the alveoli are called:


a. Alveolar macrophages
b. Pulmonary cells
c. Goblet cells
d. Chalice cells

Ans. a. Alveolar macrophages

Rationale:
• An alveolar macrophage (or dust cell) is a type of macrophage, a professional phagocyte,
found in the pulmonary alveoli, near the pneumocytes, but separated from the wall.
• Also known as ‘dust cells’, they remove dust and other debris from the alveoli by
phagocytosis. Some particles (for example, asbestos, silica) cannot be engulfed and
digested by alveolar macrophages, resulting in irreversible respiratory disease through
an accumulation of these materials within lung tissue.
National NORCET Test 6 3

5. Correctly match the type of joints.


Column-A Column-B
a. Plane synovial
b. Ball and socket
c. Hinge
d. Pivot (Trochoid)
e. Ellipsoid
f. Saddle

a. 1-d, 2-a, 3-f, 4-b b. 1-b, 2-c, 3-f, 4- e


c. 1-c, 2-f, 3-a, 4-d d. 1-b, 2-d, 3-f,4-c

Ans. b. 1-b, 2-c, 3-f, 4- e

Rationale:
The above image depicts:
• 1= Shoulder joint → Ball and socket
• 2= Elbow joint → Hinge
• 3= 1st carpometacarpal → Saddle
• 4= Metacarpophalangeal → Ellipsoid more than condylar
Types of Joint in upper limb

Joint Type
Sternoclavicular Saddle
Acromioclavicular Plane synovial
Shoulder Ball and socket
Elbow Hinge
Superior and inferior radioulnar Pivot (Trochoid)
Middle radioulnar Syndesmosis
Wrist Ellipsoid more than condylar
1st carpometacarpal Saddle (sellar)
Metacarpophalangeal Ellipsoid more than condylar
Intercarpal and midcarpal Plane synovial

6. The majority of lymph fluid in the body returns to the venous circulation via
the:-
a. Right lymphatic duct b. Cisterna chyli
c. Intestinal tract d. Thoracic duct

Ans. d. Thoracic duct


4 National NORCET Test 6

Rationale:
• After leaving the lymph node through efferent vessels, lymph travels either to another
node further into the body or to a lymph trunk, the larger vessel where many efferent
vessels converge.
• Four pairs of lymph trunks are distributed laterally around the center of the body,
along with an unpaired intestinal trunk.
• The lymph trunks then converge into the two lymph ducts, the right lymph duct and
the thoracic duct.
• These ducts take the lymph into the right and left subclavian veins, which flow into the
vena cava. This is where lymph fluid reaches the end of its journey from the interstitial
space of tissues back into blood circulation.

7. The patient is admitted to the emergency department having difficulty with


respiratory, vasomotor, and cardiac function. Which portion of the brain is
affected to cause these manifestations?
a. Medulla b. Cerebellum
c. Parietal lobe d. Wernicke’s area

Ans. a. Medulla

Rationale:
• The medulla contains the vital centers concerned with respiratory, vasomotor, and
cardiac function.
• The cerebellum maintains trunk stability and equilibrium but is not related to
respiratory or cardiac function.
• The parietal lobe interprets spatial information and controls the sensory cortex.
• Wernicke’s area is responsible for language comprehension.

8. Islets of Langerhans are rich in which part of pancreas?


a. Neck
b. Body
c. Head
d. Tail

Ans. d. Tail

Rationale:
• The islets of Langerhans are clusters of hormone-producing endocrine cells interspersed
within pancreatic exocrine tissue and it is most rich tail region of the pancreas.
• The islets of Langerhans contain three major cell types
Type of Cell Location Function
Beta Central islet Secrete insulin
Alpha Outer rim of islet Secrete glucagon
Delta Intermixed Secrete somatostatin and gastrin
National NORCET Test 6 5

9. Peripheral resistance depends on:


a. Stroke volume
b. Blood volume
c. Degree of contraction of veins
d. Diameter of arterioles

Ans. d. Diameter of arterioles

Rationale:
• Total peripheral resistance (TPR) is increased because of arteriolar vasoconstriction,
which leads to decreased blood flow to the cutaneous circulation and causes cold,
clammy skin.
• Constriction of the arterioles increases total peripheral resistance (TPR), and
constriction of the veins promotes increased venous return of blood to the heart,
both of which increase BP.

10. Which of the following statement is true about inspiratory capacity?


a. Tidal volume + volume inspired forcefully
b. Tidal volume + Volume expired forcefully
c. Volume remaining after normal expiration
d. Volume remaining after forced expiration

Ans. b. Tidal volume + Volume expired forcefully

Rationale:

Respiratory Capacities
Lung Capacity (mL)
Vital capacity (VC) 3500 IRV+TV+ERV, maximum amount of air
that can be exhaled after a maximum
inspiration
Inspiratory capacity (IC) 2500 TV+IRV, maximum amount of air that can
be inhaled after a normal expiration
Expiratory Capacity (EC) 1500 TV+ERV
Functional residual capacity 2500 RV+ERV, amount of air remaining in the
(FRC) lungs after a normal tidal expiration
Total lung capacity (TLC) 5000 RV+VC, maximum volume to which the
lungs can be expanded

Closing capacity RV+ the volume expired between the


beginning of airway closure and the RV
6 National NORCET Test 6

BIOCHEMISTRY & NUTRITION


11. Match each of the characteristics below with the source of stored energy.
Column-A Column-B
1. The energy source reserved for strenuous muscular activity a. Protein
2. The major precursor of urea in the urine b. Triacylglycerol
3. The largest amount of stored energy in the body c. Liver glycogen
4. T he primary source of carbon for maintaining blood d. Muscle glycogen
glucose levels during an overnight fast
a. 1-a, 2-d, 3-b, 4-c b. 1-d, 2-a, 3-b, 4-c
c. 1-d, 2-c, 3-b, 4-a d. 1-a, 2-b, 3-c, 4-d

Ans. b. 1-d, 2-a, 3-b, 4-c

Rationale:
• Muscle glycogen is used for energy during exercise.
• The nitrogen in amino acids derived from protein is converted to urea and excreted in
the urine. Uric acid, another excretion product that contains nitrogen, is derived from
purine bases (found in nucleic acids), not from protein.
• Adipose triacylglycerols contain the largest amount of stored energy in humans,
followed by protein (even though loss of too much protein will lead to death), muscle
glycogen, and liver glycogen.
• Liver glycogenolysis is the major process for maintaining blood glucose levels after an
overnight fast. The muscle cannot export glucose to contribute to the maintenance of
blood glucose levels, and fatty acid carbons cannot be utilized for the net synthesis of
glucose.

12. Mr. Dubey is in danger of respiratory arrest following the administration of


a narcotic analgesic. An arterial blood gas value is obtained. Nurse Priyanka
would expect the paCO2 to be which of the following values?
a. 15 mm Hg b. 30 mm Hg
c. 40 mm Hg d. 80 mm Hg

Ans. d. 80 mm Hg

Rationale:
Add normal value of paCo2
• A client about to go into respiratory arrest will have inefficient ventilation and will be
retaining carbon dioxide. The value expected would be around 80 mm Hg. All other
values are lower than expected.
• Normal Partial pressure of carbon dioxide (PaCO2) level is: 38 to 42 mm Hg.
• 15 mmHg is a low value for a client about to go into respiratory arrest.
• 30 mmHg is lower than the expected value because of inefficient ventilation.
• 40 mmHg is still less than the expected value for a client who is about to go into
respiratory arrest.
National NORCET Test 6 7

13. Ronit, aged 15 years, complains of bleeding gums. The nurse will advise him to
take a diet high in:
a. Vitamin C
b. Vitamin B
c. Vitamin A
d. Vitamin D

Ans. a. Vitamin C

Rationale:
• The hydroxylation of proline and lysine residues in collagen requires vitamin C and
oxygen.
• In the absence of vitamin C, the collagen formed cannot be appropriately stabilized
(owing, in part, to reduced hydrogen bonding between subunits due to the lack of
hydroxyproline) and is easily degraded, leading to the bleeding gums and loss of teeth.

14. Food contraindicated with CRF and spironolactone:


a. Beans
b. Papaya
c. Banana
d. Maize

Ans. c. Banana

Rationale:
• Bananas are very high in potassium. One banana has 422mg of potassium.
• Due to the feared risks of hyperkalemia, patients with CRF have been advised to avoid
consumption, or eat in moderation foods found to contain high levels (>400mg/100g)
of potassium.
• Spironolactone is a potassium-sparing diuretic (water pill) that prevents your body
from absorbing too much salt and keeps your potassium levels from getting too low.
Spironolactone is used to treat heart failure, high blood pressure (hypertension), or
hypokalemia (low potassium levels in the blood).

15. Lactose is made up of:


a. Glucose and galactose
b. Glucose
c. Glucose and fructose
d. Fructose

Ans. a. Glucose and galactose

Rationale:
• Lactose (milk sugar) contains galactose linked by β-1,4 gycosidic linkage to glucose.
• Sucrose (a component of table sugar and fruit) contains glucose and fructose residues
linked via their anomeric carbon.
8 National NORCET Test 6

Reducing Disaccharides with Free Functional Group


Disaccharide Sugar Units Linkage
Maltose Glucose + Glucose α 1, 4 linkage
Isomaltose Glucose + Glucose α 1, 6 linkage
Lactose Galactose + Glucose β 1, 4 linkage
Lactulose Galactose + Fructose α 1 β 4 linkage

Non-Reducing Disaccharides with No-Free Functional Group


Disaccharide Sugar Units Linkage
Trehalose (Sugar of insect Glucose + Glucose α 1, 1 linkage
hemolymph, yeast and Fungi)
Sucrose (Cane sugar) Galactose + Fructose α 1 β 2 linkage

MICROBIOLOGY
16. Following blood transfusion, blood bags are disposed in which color coded bin:
a. Red b. Yellow
c. Blue d. White

Ans. b. Yellow

Rationale:

Categories of Biomedical Wastes (BMW)


Cat BMW Waste Included
1 Human Anatomical waste Human tissue, organs, body parts
2 Animal waste Animal tissues, body parts, organs, carcasses, fluids, Blood
3 Microbiology and Waste from lab cultures, stocks, specimens of
Biotechnology Waste microorganisms, live or attenuated vaccine, cell cultures
(Human/ Animal), wastes from production of biological,
Toxins
4 Waste Sharps Needles, Syringes, Blades, Scalpels, Glass
5 Discarded Medicine and Outdated, contaminated and discarded drugs
Cytotoxic Drugs
6 Soiled Waste Items contaminated with blood, and fluids, including cotton,
dressings, soiled plaster casts, linen, beddings.
7 Solid waste Disposable items (except sharps) including tubing, catheters,
intravenous sets
8 Liquid Waste Waste generated from lab and washing, cleaning
housekeeping and disinfecting activities.
9 Incineration Ash Ash from incineration of any BMW
10 Chemical Waste Chemical used in disinfection (insecticides) or in production
of biological
National NORCET Test 6 9

New Biomedical Waste Management Guidelines 2015-16


• Ministry: Ministry of Environment and Forest
• Legislation: Section 6, 8 and 25 of Environment [protection Act, 1986].
• These rules shall not apply to Radioactive wastes, Hazardous chemical, Municipal solid
wastes, Lead acid batteries, Hazardous wastes, e-waste, Hazardous microorganisms,
genetically engineered microorganisms.
Category Type of Wastes Bag/ Container Treatment / Disposal
Yellow Human Anatomical Yellow non- Incineration/ Plasma pyrolysis/
waste chlorinated plastic Deep burial
Animal Anatomical bags
waste
Solid waste Incineration/ Plasma pyrolysis/
Deep burial OR
autoclaving/ microwaving /
Hydroclaving
THEN
Shredding / Mutilation
Discarded Medicine Yellow non- Incineration/Encapsulation/
Chemical Waste chlorinated plastic Plasma pyrolysis
bags
Chemical liquid Separate collection Pretreatment
Waste system leading to THEN
effluent treatment Drain
system.
Discarded linen, Yellow non- Non-chlorinated chemical
mattresses, bedding chlorinated plastic disinfection
contaminated with bags or suitable THEN
blood or body fluid packing material Incineration/ Plasma pyrolysis /
Energy recovery or Shredding /
Mutilation
Microbiology Autoclave safe Pre-treat with Non-chlorinated
Biotechnology clinical plastic bags or chemicals
laboratory waste containers THEN,
Incineration
Red Contaminated waste Red non- Autoclaving/ microwaving /
(Recyclable) chlorinated plastic Hydroclaving
bags or containers THEN
Shredding / Mutilation
THEN
Energy recovery / Plastic to diesel
or fuel oil / Road making
White Waste sharp including Puncture proof, Autoclaving/ Dry heat
(Translucent) metals leak proof, Tamper THEN
proof container Shredding / Mutilation/
Encapsulation
THEN
Iron foundries / Sanitary landfill /
Waste sharp pit
Contd…
10 National NORCET Test 6

Category Type of Wastes Bag/ Container Treatment / Disposal


Blue Glassware Cardboard boxes Sodium hypochlorite/
Metallic Body with blue colored Autoclaving/ Microwaving/
Implants marking Hydroclaving
THEN
Recycling.

17. Fungi are:


a. Prokaryotes b. Eukaryotes
c. Plant d. Animalia

Ans. b. Eukaryotes

Rationale:

Characteristics of Fungi:
• Eukaryotic Protista
• Rigid cell walls
• Has chitin, mannan and other polysaccharides
• True nuclei with nuclear membrane, paired chromosomes
• Unicellular or multicellular
• Divide sexually or asexually

18. Which of the following viruses is not a double strand linear DNA virus?
a. Poxvirus
b. Papovavirus
c. Adenovirus
d. Herpes virus

Ans. b. Papovavirus

Rationale:
• Papovaviruses are small, non-enveloped, viruses that contain circular, double-stranded
DNA. Viral particles range in diameter from 45 to 55 nm.
• Poxviruses are large, enveloped, DNA viruses that infect vertebrate and invertebrate
species and replicate entirely in the cytoplasm.
• Adenoviruses are non-enveloped, double-stranded DNA viruses.
• Herpes viruses have a unique four-layered structure: a core containing the large,
double-stranded DNA genome is enclosed by an icosapentahedral capsid which is
composed of capsomers. The capsid is surrounded by an amorphous protein coat
called the tegument. It is encased in a glycoprotein-bearing lipid bilayer envelope.

19. Modified Ziehl-Neelsen staining is used for:-


a. M. leprae b. M. bovis
c. Nocardia d. All of the above

Ans. d. All of the above


National NORCET Test 6 11

Rationale:
• The Modified Ziehl-Neelsen stain (mZN stain) is a type of differential bacteriological
stain used to identify acid-fast organisms, mainly Mycobacteria. Acid fast organisms
are those which are capable of retaining the primary stain when treated with an acid
(fast=holding capacity).
• A modified acid-fast staining method was developed for rapid detection of
Mycobacterium tuberculosis and its L forms, wherein carbol fuchsin and dioxogen were
mixed into the sputum smear. With this method, the dyeing time is shortened and
heating is not required.
• Modified Ziehl-Neelsen staining is used for
Ĕ Mycobacterium leprae
Ĕ Atypical mycobacterium
Ĕ Nocardia
Ĕ Spores
Ĕ Coccidian parasites

20. Sporulation is seen in which marked phase of bacterial growth phase in the
given image?

a. A b. B
c. C d. D

Ans. c. Stationary phase

Rationale:
Phases of bacterial growth curve

Lag phase Log phase Stationary phase Decline phase


Bacteria divide NO Yes Yes No
Bacteria death No No Yes Yes
Total count Flat Raises Raises Flat
Viable count Flat Raises Flat Falls
Special Preparatory phase, Uniformly Gram variable Produce
features Accumulation of stained, Produce: Granule, involution
metabolites, Metabolically spores, exotoxins, forms
Attains maximum size active, antibiotics,
at end of lag phase Small size bacteriocin
12 National NORCET Test 6

Fig: Bacterial growth curve

PHARMACOLOGY
21. All of the following are side effects of rifampicin except:
a. Dizziness b. Anorexia
c. Orange-colored urine d. Hypertension

Ans. d. Hypertension

Rationale:
Drugs Side Effects
Isoniazid • Isoniazid crosses blood brain barrier and is associated with neuropsychiatric
symptoms like memory loss euphoria, hallucinations etc. can be seen with
psychosis. Isoniazid inhibits pyridoxine phosphokinase required to convert
pyridoxine to pyridoxal 5 phosphate isoniazid also directly inhibits pyridoxal
5 phosphate. Since, pyridoxal 5 phosphate is require for GABA synthesis,
isoniazid thus inhibits synthesis of inhibitory neurotransmitter GABA.
• Isoniazid can inhibit delta-amino levulinate synthase, which leads to
decrease in haem synthesis and sideroblastic anemia.
• Arthritis involving the upper limb known as shoulder syndrome can be seen.
• Gynecomastia

Mnemonics: “CHANGE”
• Change in memory
• Hepatotoxic, Hallucinations
• Anemia, Arthritis
• Neuropathy
• Gynecomastia
• Euphoria, Epilepsy
Contd…
National NORCET Test 6 13

Drugs Side Effects


Rifampicin • Most common side-effect is nausea and vomiting followed by rash, fevers
etc. Rash caused by anti TB drugs are non-petechial rash.
• At intermittent doses it can cause flu like symptoms, respiratory syndrome
(shortness of breath), cutaneous reactions (flushing and itching),
thrombocytopenic purpura and abdominal reactions (pain and nausea).
• Reddish orange discoloration of urine, saliva, contact lenses, tears etc. can
be seen and is known as “Red man syndrome”.
Pyrazinamide • Pyrazinamide is most hepatotoxic of all first line drugs. Its toxic metabolites
are excreted by kidney and hence dose reduction is required in renal failure.
The order of hepatotoxicity of first line drugs are Pyrazinamide > Isoniazid
> Rifampicin.
• It can also cause asymptomatic hyperuricemia and arthralgia,
Ethambutol • Ethambutol can cause optic neuritis and red-green color blindness (green >
red). That is why it is avoided in children.
• Other side effect like hyperuricemia (lesser than pyrazinamide).
• As Ethambutol is excreted by kidney, its dose should decrease in renal
failure.

22. Withdrawal symptoms do NOT usually occur in case of which of the following
medications?
a. Monoamine oxidase inhibitors
b. Non-steroidal anti-inflammatory drugs
c. Selective serotonin reuptake inhibitors
d. Tetracyclic anti-depressants

Ans. b. Non-steroidal anti-inflammatory drugs

Rationale:
• Monoamine Oxidase Inhibitors (MAO inhibitors) are used to treat depressive
disorders, including atypical depression (presenting with psychotic, polyphagic, or
phobic features). MAO inhibitors are used less often since the introduction of SSRIs
because of their dangerous interaction with foods containing tyramine.
• Selective Serotonin Reuptake Inhibitors (SSRIs) are the current first-line treatment
for depressive and anxiety disorders. They prevent reuptake of serotonin by the
presynaptic terminal, allowing for increased availability of serotonin to the postsynaptic
membrane.
• Tricyclic Antidepressants (TCAs) can be used to treat chronic pain, major depression,
and anxiety disorders. Historically, TCAs have been used in children for the treatment
of enuresis.
• The above three drugs have withdrawal effect; except Non-steroidal anti-inflammatory
drugs (NASIDs).

23. In acetaminophen toxicity, nurse observe:


a. Transaminase levels b. Troponin levels
c. Complete blood count d. EEG changes

Ans. a. Transaminase levels


14 National NORCET Test 6

Rationale:
• Acetaminophen used as Antipyretic and analgesic but no effect on inflammation.
• Side effects Acetaminophen: Significant hepatic necrosis can occur with high doses,
as hepatic metabolism of acetaminophen creates a highly toxic intermediate called
N-acetyl-p-benzoquinone imine (NAPQI).
• In acetaminophen, poisoning the extreme AST and ALT elevation (AST and ALT ≥
5000) is seen. So, the nurse will check transaminase level.
• N-acetylcysteine is also used as an antidote for acetaminophen overdose.

24. Patient is receiving continuous salbutamol nebulization therapy. What should


a nurse assess for?
a. S. calcium
b. S. potassium
c. S. magnesium
d. S. sodium

Ans. b. S. potassium

Rationale:
• Salbutamol is short acting β2 agonists (SABA) used as a bronchodilator.
• Side effect of β2 agonists is hypokalemia. So, the nurse checks the patient’s serum
potassium level.
• Other Side effects of β2 agonists are:
Ĕ Tremors are most common side-effect seen due to β2 receptor stimulation in
skeletal muscles.
Ĕ Palpitation, hyperglycemia, ventilation perfusion mismatch and QT prolongation.

25. Effect of vasodilator includes:


a. Hypotension
b. Hypertension
c. Diuresis
d. Sweating

Ans. a. Hypotension

Rationale:
• Endothelial cells can also release vasodilators (nitric oxide, prostacyclin) and
vasoconstrictors (endothelin).
• Vasodilators reduce arterial resistance or increase venous capacitance; the net effect is
a reduction in vascular pressure. In response to failures of pump function, sympathetic
tone increases during the resting state, causing excessive venoconstriction and
ultimately reduces cardiac output. Thus, vasodilators can be effective in CHF, and they
are particularly useful when heart failure is associated with hypertension, congestive
cardiomyopathy, mitral or aortic insufficiency, or ischemia.
• Vasodilators are used to treat severe, decompensated CHF refractory to diuretics and
digitalis.
National NORCET Test 6 15

26. Which of the following drugs is used for the treatment of nocturnal enuresis?
a. Imipramine b. Chlorpromazine
c. Trazodone d. Sertraline

Ans. a. Imipramine

Rationale:
• Enuresis refers to night-time urinary incontinence ≥ 2 times/week for ≥ 3 months in
person > 5 years old.
• TCAs like imipramine are used infrequently to suppress enuresis in children (over age
6) and adults.
• First-line treatment: Behavioral modification (e.g., scheduled voids, night-time fluid
restriction) and positive reinforcement.
• Desmopressin (ADH analog) is a treatment for central DI and nocturnal enuresis.

27. The maximum dose of atropine which can be given in bradycardia is:
a. 1 mg b. 3 mg
c. 5 mg d. 10 mg

Ans. b. 3 mg

Rationale:
• Atropine is the first line medication for the treatment of bradycardia. The administration
of atropine typically causes an increase in heart rate. This increase in the heart rate
occurs when atropine blocks the effects of the vagus nerve on the heart.
• The dosing for Atropine is 0.5 mg IV every 3-5 minutes as needed, and the maximum
total dosage for administration is 3 mg.
• Atropine should be avoided with bradycardia caused by hypothermia.

28. Which of the following is NOT an adverse effect of morphine?


a. Euphoria b. Respiratory stimulation
c. Nausea d. Histamine release

Ans. b. Respiratory stimulation

Rationale:
• Morphine is used for analgesia in severe preoperative and postoperative pain, as well
as for the pain of terminal illness; it is used to treat the visceral pain of trauma, burns,
cancer, acute myocardial infarction (MI), and renal or biliary colic.
• Morphine is usually given parenterally, but it can be given orally or rectally.
• Adverse effects and contraindications of morphine:
Ĕ Respiratory depression
Ĕ Constipation
Ĕ Postural hypotension
Ĕ Nausea and vomiting
Ĕ Pneumonia
Ĕ Sedative activity with drowsiness
16 National NORCET Test 6
Ĕ Pain from biliary or urinary tract spasm
Ĕ Urine retention
Ĕ Miosis

29. Method used to give Haloperidol injection is:


a. Z track b. M track
c. N track d. W track

Ans. a. Z track

Rationale:
• Haloperidol is an antipsychotic agent used in acute psychotic attacks and for
the treatment of schizophrenia. It is a dopamine-receptor antagonist that acts
predominately at the dopamine D2 receptor.
• When a medication is injected directly into muscle, it is called an intramuscular
injection (IM).
• The Z-track method is a type of IM injection technique of injecting medication into
a large muscle using a needle and syringe. This method seals the medication deeply
within the muscle and allows no exit path back into the subcutaneous tissue and skin.
• It is also the method of choice when giving I.M. medications that are very irritating to
the tissue, such as haloperidol or Vistaril.
• The Z-track method of intra-muscular (I.M.) injection is used primarily when
giving dark-colored medication solutions, such as iron solutions, that can stain the
subcutaneous tissue or skin.

30. Tetracycline is used as prophylaxis for:-


a. Leptospirosis b. Brucellosis
c. Cholera d. Meningitis

Ans. a. Leptospirosis

Rationale:
• Tetracyclines are derivatives of naphthacene carboxamide.
• Tetracyclines bind reversibly to the 30S subunit of bacterial ribosomes. This prevents
the binding of aminoacyl tRNA to the acceptor site on the mRNA–ribosome complex
and addition of amino acids to the growing peptide, thus inhibiting bacterial protein
synthesis; these agents are bacteriostatic.
• Tetracyclines are active against both gram-negative and gram-positive organisms, but
the use of these agents is declining because of increased resistance and the development
of safer drugs.
• Tetracyclines are used predominantly for the treatment of rickettsial infections,
including Rocky Mountain spotted fever, cholera, Lyme disease, and infections caused
by Chlamydia spp. and Mycoplasma pneumoniae.
• Leptospirosis—Flu-like symptoms, myalgias (classically of calves), jaundice,
photophobia with conjunctival suffusion (erythema without exudate). Prevalent
among surfers and in tropics (e.g., Hawaii).
• Tetracycline is used as prophylaxis for Leptospirosis and plague.
National NORCET Test 6 17

31. The antidote used in iron toxicity is:


a. N-Acetylcysteine b. Deferoxamine
c. Atropine d. Charcoal

Ans. b. Deferoxamine

Rationale:
Poisons Antidotes
Iron , Aluminium Desferrioxamine
Paracetamol Acetyl cysteine , Methionine
Cholinergic agents Atropine
Ipecac-induced emesis Charcoal

32. Which is the most important route of excretion of drugs?


a. Kidney
b. Saliva and sweat
c. Exhaled air
d. Feces

Ans. a. Kidney

Rationale:
• Routes of excretion may include urine, feces (e.g., unabsorbed drugs and drugs secreted
in bile), saliva, sweat, tears, milk (with possible transfer to neonates), and lungs (e.g.,
alcohols and anesthetics). Any route may be important for a given drug, but the kidney
is the major site of excretion for most drugs.

33. Urination in the human subject is decreased by:-


a. AChase inhibitors
b. Muscarinic agonists
c. Nicotinic agonists
d. Muscarinic antagonists

Ans. d. Muscarinic antagonists

Rationale:
• Urinary retention is a well-known adverse effect of drugs that have antagonist effects
on muscarinic receptors. In addition to the prototypic drug atropine, M blockers
include drugs used in Parkinson disease, such as benztropine.
• Acetylcholine directly and AChE inhibitors (Edrophonium, Physostigmine) indirectly
activate M receptors in the GU system, causing bladder contraction with voiding and
incontinence.
• Activation of nicotinic receptors in ANS ganglia would lead to the stimulation of PANS
functions.
18 National NORCET Test 6

34. Which of the following is the drug of choice for chemoprophylaxis of cholera?
a. Tetracycline b. Macrolides
c. Furazolidone d. Cotrimaxazole

Ans. a. Tetracyclines

Rationale:
• Tetracycline is the drug of choice for chemoprophylaxis of cholera.
• It should be given for
Ĕ 3-days period for twice-daily dose of 500 mg for adults.
Ĕ 125 MG for children aged 4-13 years.
Ĕ 50 mg for children aged 0-3 years.
• Alternatively, single dose of doxycycline may be used for chemoprophylaxis. The dose
is 300 mg for adults and 6 mg/Kg for children under 15 years.

35. A nurse provides IV administration to a patient, the initial rates of drug


distribution to different tissues of the patient depend primarily on which of the
following parameters?
a. Fat content of the tissues
b. Active transport of the drug out of different cell types
c. Blood flow to the tissues
d. Degree of ionization of the drug in the tissues

Ans. c. Blood flow to the tissues

Rationale:
• The initial rate of distribution of a drug to a tissue depends primarily on the rate of
blood flow to that tissue.
• At longer times, however, a drug may undergo redistribution among various tissues,
e.g., a very lipophilic drug may become concentrated in adipose tissue with time.

COMMUNITY HEALTH NURSING


36. There are a number of risk factors associated with coronary artery disease.
Which of the following is a modifiable risk factor?
a. Obesity b. Heredity
c. Gender d. Age

Ans. a. Obesity

Rationale:
• Obesity is an important risk factor for coronary artery disease that can be modified by
improved diet and weight loss.
• Family history of coronary artery disease, male gender, and advancing age increase risk
but cannot be modified.
National NORCET Test 6 19

37. Who is the father of epidemiology?


a. Fracastorius
b. John Snow
c. Panacea
d. Hippocrates

Ans. b. John Snow

Rationale:
• John Snow: Father of Epidemiology
• Fracastorius: Founder of Epidemiology
• Panacea: Goddess of medicine
• Hippocrates: Father of medicine

38. Occupational exposure that may cause sterility in females:-


a. Aniline b. Lead
c. Radon d. Nickel

Ans. c. Radon

Rationale:
• Ionizing radiations: X-rays and radioactive isotopes (e.g. cobalt 60, phosphorus 32)
can lead to genetic changes, malformations, cancer, leukemia, depilation, ulceration,
sterility and in extreme cases death.
• Maximum permissible level of occupational exposure set by international commission
of Radiological Protection is 5rem per year to the whole body.
• Lead exposure : Plumbism
• Nickel exposure: Lung cancer, nasal sinus cancer
• Anilines exposure: Leukemia and bladder cancer

39. This walk through tunnel for COVID 19 uses which of the following?

a. 5% cresol b. Cetrimide
c. 1% sodium hypochlorite d. 2% glutaraldehyde

Ans. c. 1% sodium hypochlorite


20 National NORCET Test 6

Rationale:
All indoor areas such as entrance lobbies, corridors and staircases, escalators, elevators,
security guard booths, office rooms, meeting rooms, cafeteria should be mopped with a
disinfectant with 1% sodium hypochlorite or phenolic disinfectants.

40. Which is not a demographic indicator:-


a. Social mobility b. Malnutrition
c. Immigration d. Emigration

Ans. b. Malnutrition

Rationale:
• The demographic indicators are:-
Ĕ Births
Ĕ Deaths
Ĕ Social mobility (change in the socioeconomic status)
Ĕ Migrations
Ĕ Female literacy
Ĕ Age at the time of marriage
Ĕ Duration of married life
Ĕ Spacing of children

41. All of the following are true about ASHA; except:


a. She is trained for a period of 21 days
b. Works with the village Health and Sanitation Committee
c. Acts as depot holder for IFA , ORS
d. She is in the age group 25-45 years.

Ans. a. She trained for a period of 21 days

Rationale:
• The induction training of ASHA is for 23 days in 5 episodes spread over 1 year.
• After 6 months of her functioning in the village, she is sensitized on HIV/AIDs, STI,
RTI, prevention and referrals.
• She is also trained on newborn care.
• The central government bars the cost of training, incentives and medical kits.
• ASHA is the depot holder for essential drugs and consumables like; ORS, contraceptives,
IFA.
• ASHA must be primarily a woman resident of the village-‘Married/Widow/Divorced/
Separated’ and preferably in the age group of 25-45 years.

42. Blood bags are discarded in:


a. Yellow category b. Red category
c. White Category d. Blue category

Ans. a. Yellow category


National NORCET Test 6 21

Rationale:

New Biomedical Waste Management Guidelines-2016


Ĕ Ministry: Ministry of Environment and Forest
Ĕ Legislation: Section 6, 8 and 25 of Environment protection Act, 1986.
Ĕ These rules shall not apply to Radioactive wastes, Hazardous chemical, Municipal
solid wastes, Lead acid batteries, Hazardous wastes, e-waste, Hazardous
microorganisms, genetically engineered microorganisms.
Category Type of Wastes Bag/ Container Treatment / Disposal
YELLOW Human Anatomical waste Yellow non- Incineration/ Plasma pyrolysis/
Animal Anatomical waste chlorinated plastic Deep burial
bags
Solid waste Incineration/ Plasma pyrolysis/
Deep burial OR
autoclaving/ microwaving /
Hydroclaving
THEN
Shredding / Mutilation
Discarded Medicine Yellow non- Incineration/Encapsulation/
Chemical Waste chlorinated plastic Plasma pyrolysis
bags
Chemical liquid Waste Separate collection Pretreatment
system leading to THEN
effluent treatment Drain
system.
Discarded linen, Yellow non- Non-chlorinated chemical
mattresses, bedding chlorinated plastic disinfection
contaminated with blood bags or suitable THEN
or body fluid packing material Incineration/ Plasma
pyrolysis / Energy recovery or
Shredding / Mutilation
Microbiology Autoclave safe Pre-treat with Non-chlorinated
Biotechnology clinical plastic bags or chemicals
laboratory waste containers THEN,
Incineration
RED Contaminated waste Red non- Autoclaving/ microwaving /
(Recyclable) chlorinated plastic Hydroclaving
bags or containers THEN
Shredding / Mutilation
THEN
Energy recovery / Plastic to
diesel or fuel oil / Road making
WHITE Waste sharp including Puncture proof, Autoclaving/ Dry heat
(Translucent) metal’s leak proof, Tamper THEN
proof container Shredding / Mutilation/
Encapsulation
THEN
Iron foundries / Sanitary
landfill / Waste sharp pit
Contd…
22 National NORCET Test 6

Category Type of Wastes Bag/ Container Treatment / Disposal


Blue Glassware Cardboard boxes Sodium hypochlorite/
Metallic Body Implants with blue colored Autoclaving/ Microwaving/
marking Hydroclaving
THEN
Recycling

Remember:
Category Types of Waste
YELLOW • Anatomical, animal, placenta, fetus, soiled waste, discarded linen, beddings
• Cytotoxic drugs (special mention – CYTOTOXIC waste)
• Microbiological, blood bags, lab waste, expired medicines
• Infectious liquid waste, body secretions, liquid chemical waste from lab,
disinfectants, X-ray film liquid
RED Foley’s, Uro-bag, RT, iv bottle, gloves, syringe without needle, vacutainer
BLUE Broken glass, empty vials, metallic body implants
WHITE Needles, scalpel, syringe with fixed needle, LP needle, suture needle, sharps

43. In the emergency department, the nurse assesses a child suspected of having
measles. Which of these infection control precautions should the nurse initiate?
a. Airborne precautions
b. Contact precautions
c. Reverse isolation
d. Droplet precautions

Ans. a. Airborne precautions

Rationale:
• Airborne precautions are used to prevent the transmission of infectious agents that
remain infectious over long distances when suspended in the air.
• Disease requiring airborne precautions include, but are not limited to: Measles, severe
acute respiratory syndrome (SARS), varicella (chicken pox), and mycobacterium
tuberculosis.
• Droplet infection: Respiratory infections, Whooping cough and TB
• Reverse isolation is theprocedure designed to protect a patient from infectious
organisms carried by staff, patients or other patients. The organisms are typically
spread by droplets in the air or on equipment. All staff should wash their hands prior
to entering the room. Hospitalized patients with neutropenia are commonly placed in
reverse isolation intended to protect them from acquiring serious infections.
National NORCET Test 6 23

44. All of the followings are true about the given below image; except:-

a. It is established in 1919
b. It helps poorer nations to develop their human and natural resources
c. Headquarters of the agency is in Geneva, Switzerland
d. It contributes to establishpeace by promoting social justice.

Ans. b. It helps poorer nations to develop their human and natural resources

Rationale:
• The above given image is of International Labour Organization (ILO). All of the
following options are true regarding ILO, except option B (Helps poorer nations to
develop their human and natural resources); which is the function of United Nations
Development Programme (UNDP).

International Labour Organisation


• It is established in 1919, to improve working and living conditions of working
population all over the world.
• Headquarters of the ILO is in Geneva, Switzerland.
• ILO contributes to establish peace by promoting social justice.
• Promotion of economic and social stability.

45. The burse gives a lecture at the senior center about preventive health activities
for people over the age of 60 years. The nurse tells the people that the center
for Disease Control and Prevention (CDC) now recommended which of the
following vaccines for this age group?
a. Pertussis (Whopping cough) b. Shingles (herpes zoster)
c. Meningitis d. Diphtheria

Ans. b. Shingles

Rationale:
• The Shingles vaccine reduces the risk of shingles by about half and the risk of
postherpetic neuralgia by two-thirds.
• Pertussis and diphtheria vaccine is given much earlier in life.
• The CDC recommended that college freshmen living in dormitories get the meningitis
vaccine, but this is likely to apply to those over age 60.
24 National NORCET Test 6

46. Increased occurrence of Road traffic accidents on weekends is described by


which term:-
a. Cyclic trend
b. Periodic trend
c. Seasonal trends
d. Transitional trends

Ans. a. Cyclic trend

Rationale:
• Cyclic trend refers to occurrence of a disease/event in cycles spread over short period
of time (i.e. Days, Week, Month, Year)
• In case of infectious diseases, it is due to build -up or susceptible in the “herd”.
• Measles (every 2-3 years in pre vaccination era) Rubella (every 6-9 years), Influenza
pandemics (7-10 years due to antigenic variations).
• Influenza pandemics occur at intervals of 7-10 years due to antigenic variations.
• Non-infectious conditions may also show cyclic trend e.g. Road accidents on week-
ends (Saturdays).
• Seasonal trend is due to favorable environmental conditions (e.g., Temperature,
Humidity, Rainfall etc.).
• Both infectious (Measles, Varicella in early spring. ARI in winter, GI infection in
summer) and noninfectious (sunstroke, hay fever, snakebite) disease show seasonal
trend.

47. The ability of a screening test to distinguish those who have disease from those
who don’t have is called:
a. Reliability
b. Validity
c. Precision
d. Acceptability

Ans. b. Validity

Rationale:
• Validity (Accuracy) is the degree of closeness of a measured/ calculated quantity to its
actual value.
• It has 2 components - Sensitivity and Specificity.
• Precision is also known as repeatability or reliability or reproducibility. It means the
test must give consistent results when repeated more than once on the same individual
or material, under similar conditions.

48. Hand washing to prevent nosocomial infection is an example of:-


a. Primordial prevention b. Primary prevention
c. Secondary prevention d. Tertiary prevention

Ans. b. Primary prevention


National NORCET Test 6 25

Rationale:
• Primordial prevention: Prevention of the occurrence of risk factors (Individual and
mass education)
• Primary prevention: Prevention of the occurrence of disease (Health promotion and
Specific protection)
• Secondary prevention: Prevention of the occurrence of complication of disease (Early
diagnosis and Treatment)
• Tertiary prevention: Prevention of the occurrence of disability or fatality due to a
complication from the disease.

Level of prevention and Mode of intervention


Level of prevention Mode of intervention Examples
Primordial Individual and Mass • Discourage adoption of high risk behavior
education in a population where it does not exist.
E.g. Smoking, Drinking in societies where
it is not prevalent yet.

Primary Health promotion • Health education


• Environment modification, e.g. Provision
of safe water, sanitary latrines, control of
insects and rodents
• Life style and behavioral changes, e.g.
Exercise and medication
• Improvement in overall socioeconomic
status of the population
• Promotion of breastfeeding
• Promotion of small family norms
• Marriage counselling
Specific protection • Immunization
• Chemoprophylaxis
• Nutrition supplement, e.g. IFA, Vitamin
A, Iodine
• Avoidance of allergens
• Protection against occupational hazards,
accident, carcinogen
• Control of air pollution, noise control,
chlorination of water
• Use of helmets and seat belts to protect
against head injuries
• Use of mosquito net
• Personal hygiene and environmental
sanitation
• Contraception
Contd…
26 National NORCET Test 6

Level of prevention Mode of intervention Examples


Secondary Early diagnosis Screening (Pap smear, self-breast
examination, mammography)active case
search, medical examination of school
children, infants and young children,
industrial workers
Treatment • DOTS
• MDT for leprosy
• Dental filling and tooth extraction
Tertiary Disability limitation • Resting affected limb in neutral position
to prevent stress in paralyzed muscle
• Plaster cast to a patient with colles
fracture
• Rest, morphine, oxygen and
streptokinase to a patient of acute MI.
Rehabilitation • School for blind
• Reconstructive surgery
• Arranging for schooling of child suffering
from PRPP
• Provision of hearing aids, artificial limb,
and calipers
• Restoration of capacity to earn a
livelihood (Vocational)
• Restoration of family and social relation
(Social)
• Restoration of personal dignity and
confidence (Psychological)

49. The amount of nutrition provided in the given below program is:

a. 6gm of protein + 450Kcal of energy


b. 6gm of protein + 600Kcal of energy
c. 12gm of protein + 450Kcal of energy
d. 12gm of protein + 600Kcal of energy

Ans. c. 12gm of protein + 450Kcal of energy


National NORCET Test 6 27

Rationale:
• The given below programme depicts Mid-Day Meal scheme.
• The main objective of the programme is to enhance school enrolment, retention and
attendance of children along with providing nutritional support to school children.
• It should supply at least 1/3rd of total energy requirement and 1/2 of the protein (12
gm of protein + 450Kcal of energy) needed.

50. Microspora faeni causes:-


a. Bagassosis b. Farmer’s lung
c. Suberosis d. Sequousis

Ans. b. Farmer’s lung

Rationale:
• Farmer’s lung: Due to inhalation of Moldy hay or grain dust. Thermophilic
actinomycete (Micropolyspora faeni) is the causative agent. Acute illness is
characterized by general and respiratory symptoms and physical signs. Repeated attacks
can cause pulmonary fibrosis, inevitable pulmonary damage and Corpulmonale.
• Bagassosis is an occupational disease of the lung caused by inhalation of bagasse or
sugarcane dust. Thermophilic actinomycete (Thermoactinomyces sacchari) is the
causative agent.
• Suberosis is a type of hypersensitivity pneumonitis usually caused by the fungus
Penicillium glabrum (formerly called Penicillum frequentans) from exposure to moldy
cork dust.

PEDIATRICS
51. The 6-month-old client with a ventral septal defect is receiving Digitalis for
regulation of his heart rate. Which finding should be reported to the doctor?
a. Heart rate of 60 bpm b. Blood pressure of 126/80
c. Respiratory rate of 30 per minute d. Blood glucose of 110 mg/dL

Ans. a. Heart rate of 60 bpm

Rationale:
• A heart rate of 60 in the baby should be reported immediately. The dose should be held
if the heart rate is below 100 bpm. Digoxin has vagomimetic effects on the AV node. By
stimulating the parasympathetic nervous system, it slows electrical conduction in the
atrioventricular node, therefore, decreases the heart rate.
• The physician must request regular electrocardiograms and blood work to assess for
renal function, and electrolytes required for close monitoring.
• Electrocardiogram changes seen with digoxin demonstrate a downsloping ST-segment
depression, also known as a “reverse check” sign. The ST segments may appear
“scooped” without abnormal Q waves or T wave inversions.
28 National NORCET Test 6

52. A nurse is counseling the mother of a newborn infant with hyperbilirubinemia.


Which of the following instructions by the nurse is not correct?
a. Follow up with the infant’s physician within 72 hours of discharge for a recheck of the
serum bilirubin and exam.
b. Keep the baby quiet and swaddled, and place the bassinet in a dimly lit area.
c. Continue to breastfeed frequently, at least every 2-4 hours.
d. Watch for signs of dehydration, including decreased urinary output and changes in skin
turgor.

Ans. b. Keep the baby quiet and swaddled, and place the bassinet in a dimly lit area

Rationale:
• An infant discharged home with hyperbilirubinemia (newborn jaundice) should be
placed in a sunny rather than dimly lit area with skin exposed to help process the
bilirubin.
• Phototherapy is started based on risk factors and the serum bilirubin level on the
nomogram. Bilirubin absorbs light optimally in the blue-green range (460 to 490 nm)
and is either photo isomerized and excreted in the bile or converted into Lumirubin
and excreted in the urine. During phototherapy, the eyes of the newborn must be
covered, and the maximum body surface area exposed to the light.
• A recheck of the serum bilirubin and a physical exam within 72 hours will confirm that
the level is falling and the infant is thriving and is well hydrated. Bilirubin levels may be
assessed using a transcutaneous measurement device or taking blood for total serum
or plasma level determination.
• Frequent feedings will help to metabolize the bilirubin. Breast milk jaundice occurs
late in the first week, peaks in the second, and usually resolves by 12 weeks of age.
• Signs of dehydration, including decreased urine output and skin changes, indicate
inadequate fluid intake and will worsen hyperbilirubinemia. Breastfeeding jaundice,
also known as breastfeeding failure jaundice, occurs in the first week of life and is
due to failure of adequate intake of breast milk leading to dehydration and sometimes
hypernatremia.

53. A non-immunized child appears at the clinic with a visible rash. Which of the
following observations indicates the child may have Rubeola (measles)?
a. The lesions have a “teardrop on a rose petal” appearance
b. Small blue-white spots are visible on the oral mucosa
c. There is low-grade fever
d. The rash begins on the trunk and spreads outward

Ans. b. Small blue-white spots are visible on the oral mucosa

Rationale:
• Koplik’s spots are small blue-white spots visible on the oral mucosa and are characteristic
of measles infection.
• Most cases show the characteristic Koplik spots of the disease, located in the buccal
mucosa at the height of the second molar, and appear two to three days before the rash
and disappear on the third day.
National NORCET Test 6 29

• “Teardrop on a rose petal” refers to the lesions found in varicella (chickenpox).


• High fever is often present. The primary or prodromal phase lasts four to six days and
is characterized by the presence of high fever, malaise, coryza, conjunctivitis, palpebral
edema, and dry cough.
• The body rash typically begins on the face and travels downward. The second phase,
the eruptive, is characterized by the appearance of a maculopapular rash, initially fine
that subsequently becomes confluent. The rash begins behind the auricle and along
the hair implantation line, and extends downward to the face, trunk, and extremities.

54. The nurse is teaching basic infant care to a group of first-time parents. The
nurse should explain that a sponge bath is recommended for the first 2 weeks
of life because:
a. The chances of chilling the baby outweighs the benefits of bathing
b. New parents need time to learn how to hold the baby
c. Newborn skin is easily traumatized by washing
d. The umbilical cord needs time to separate

Ans. d. The umbilical cord needs time to separate

Rationale:
• The umbilical cord needs time to dry and fall off before putting the infant in the
tub. The first bath will be a sponge bath. Pick a warm room with a flat surface, like
a bathroom or kitchen counter, a changing table, or a bed. Cover the surface with a
thick towel. Make sure the room temperature is at least 75 degrees Fahrenheit, because
babies chill easily.
• Option A: Although these statements might be important, they are not the primary
answer to the question. The World Health Organization (WHO) recommends delaying
baby’s first bath until 24 hours after birth—or waiting at least 6 hours if a full day isn’t
possible for cultural reasons. Babies who get baths right away may be more likely to
become cold and develop hypothermia. The minor stress of an early bath can also
make some babies more likely to have a drop in blood sugar (hypoglycemia).
• Option B: Taking the baby away for a bath too soon can interrupt skin-to-skin care,
mother-child bonding, and early breastfeeding success. One study showed a 166%
increase in hospital breastfeeding success after implementing a 12-hour delay in the
baby’s first bath compared to those bathed within the first couple hours.
• Option C: Gentle sponge baths are perfect for the first few weeks until the umbilical
cord falls off, the circumcision heals, and the navel heals completely. Once the umbilical
cord falls off, and the circumcision and the navel are completely healed, it’s time to try
a tub bath.
30 National NORCET Test 6

55. A 4-day-old neonate is under phototherapy in NICU. The child has shrill cry
with neck retraction. Diagnosis is:

a. Kernicterus b. Tetanus
c. Erbs’ palsy d. Tetany

Ans. a. Kernicterus

Rationale:
• The picture shows an infant with hypertonia of extensor muscles (as indicated by
opisthotonus arching of the spine and retrocollis - hyperextension of neck) which in
the setting of hyperbilirubinemia is suggestive of kernicterus.
• Kernicterus is a pathologic diagnosis and refers to yellow staining of the brain (most
commonly in basal ganglia and cranial nerves) by bilirubin together with evidence of
neuronal injury.

56. 7-year-old Damon has cystitis; which of the following would Nurse Elena
expect when assessing the child?
a. Dysuria b. Costovertebral tenderness
c. Flank pain d. High fever

Ans. a. Dysuria

Rationale:
• Option A: Dysuria is a symptom of a lower urinary tract infection (UTI) such as
cystitis.
• Options B, C, & D: Costovertebral tenderness, flank pain, and high fever are signs and
symptoms of pyelonephritis, an upper UTI.

57. A 4-month-old is brought to the well-baby clinic for immunization. In addition


to the DPT and polio vaccines, the baby should receive:
a. MMR
b. Hib Titer
c. Hepatitis B vaccine
d. Mumps vaccine

Ans. b. Hib Titer


National NORCET Test 6 31

Rationale:
• The Haemophilus influenza vaccine is given at 4 months with the polio vaccine.
• It protects the child from Hib disease, which can cause lifelong disability and be deadly;
protects the child from the most common type of Hib disease, meningitis.
• MMR stands for measles, mumps and rubella vaccine, which is given at 9 months old.
• Hepatitis B vaccine is given immediately after birth and for children up to 18 years.
Hepatitis B vaccine is usually given as 2, 3, or 4 shots. Infants should get their first dose
of hepatitis B vaccine at birth and will usually complete the series at 6 months of age
(sometimes it will take longer than 6 months to complete the series). Children and
adolescents younger than 19 years of age who have not yet gotten the vaccine should
also be vaccinated.
• Mumps vaccine is the best way to decrease the risk of getting mumps. It is usually
given as part of a combination vaccine that protects against three diseases: measles,
mumps, and rubella (MMR). This vaccine is only licensed for use in children who are
12 months through 12 years of age.

58. A nurse in the emergency department is observing a 4-year-old child for signs
of increased intracranial pressure after a fall from a bicycle, resulting in head
trauma. Which of the following signs or symptoms would be cause for concern?
a. Repeated vomiting
b. Bulging anterior fontanel
c. Inability to read short words from a distance of 18 inches
d. Signs of sleepiness at 10 PM

Ans. a. Repeated vomiting

Rationale:
• Increased pressure caused by bleeding or swelling within the skull can damage delicate
brain tissue and may become life threatening. Repeated vomiting can be an early sign
of pressure as the vomit center within the medulla is stimulated. Clinical suspicion
for intracranial hypertension should be raised if a patient presents with the following
signs and symptoms: headaches, vomiting, and altered mental status varying from
drowsiness to coma.
• The anterior fontanelle is closed in a 4-year-old child. The average closure time of the
anterior fontanelle ranges from 13 to 24 months. Infants of African descent statically
have larger fontanelles that range from 1.4 to 4.7 cm, and in terms of sex, the fontanelles
of male infants will close sooner compared to female infants.
• The average 4-year-old child cannot read yet, so this too is normal. Most children learn
to read by 6 or 7 years of age. Some children learn at 4 or 5 years of age. Even if a child
has a head start, she may not stay ahead once school starts. The other students most
likely will catch up during the second or third grade.
• Evidence of sleepiness at 10 PM is normal for a four year old. Newborns spend most
of their day sleeping, and they only wake up to be fed, on the other hand, 1-year-old
infants sleep for 10 to 12 hours at night without waking. The coordination between
biological rhythm and sleep-wake cycle develops over the first six months of life.
32 National NORCET Test 6

59. Rohon, age 8, is admitted with rheumatic fever. Which clinical finding indicates
to the nurse that Rohon needs to continue taking the salicylates he had received
at home?
a. Chorea b. Polyarthritis
c. Subcutaneous nodules d. Erythema marginatum

Ans. b. Polyarthritis

Rationale:
• Chorea is the restless and sudden aimless and irregular movements of the extremities
suddenly seen in persons with rheumatic fever, especially girls.
• Polyarthritis is characterized by swollen, painful, hot joints that respond to salicylates.
• Subcutaneous nodules are non-tender swellings over bony prominences sometimes
seen in persons with rheumatic fever.
• Erythema marginatum is a skin condition characterized by non-pruritic rash, affecting
trunk and proximal extremities, seen in persons with rheumatic fever.

60. A child can walk with one hand held at?

a. 8 months b. 9 months
c. 11 months d. 18 months

Ans. c. 11 months

Rationale:
• A child can stand holding on to object with both hands at 8 months.
• At 10-11 months can around holding on the furniture with one hand.
• At 11-12 months walks with one hand held.
• At 18 months can run.

61. A child is admitted to the hospital with an uncontrolled seizure disorder. The
admitting physician writes orders for actions to be taken in the event of a
seizure. Which of the following actions would not be included?
a. Restrain the patient’s limbs
b. Notify the physician
c. Administer rectal diazepam
d. Position the patient on his/her side with the head flexed forward

Ans. a. Restrain the patient’s limbs


National NORCET Test 6 33

Rationale:
• During a witnessed seizure, nursing actions should focus on securing the patient’s
safety and curtailing the seizure. Restraining the limbs is not indicated because strong
muscle contractions could cause injury. Use and pad side rails with the bed in lowest
position, or place the bed up against the wall and pad floor if rails are not available or
appropriate.
• The nurse should notify the physician in the event of a seizure so he could prescribe
the correct medication. Ascertain knowledge of various stimuli that may precipitate
seizure activity. Alcohol, various drugs, and other stimuli (loss of sleep, flashing lights,
prolonged television viewing) may increase brain activity, thereby increasing the
potential for seizure activity.
• Rectal diazepam may be a treatment ordered by the physician, who should be notified
of the seizure. Diazepam may be used alone (or in combination with phenobarbital) to
suppress status seizure activity.
• A side-lying position with head flexed forward allows for drainage of secretions and
prevents the tongue from falling back, blocking the airway. Turn head to side and
suction airway as indicated. Insert plastic bite blocks only if the jaw is relaxed. Helps
maintain airway patency and reduces the risk of oral trauma but should not be “forced”
or inserted when teeth are clenched because dental and soft-tissue damage may result.
Note: Wooden tongue blades should not be used because they may splinter and break
in the patient’s mouth.

62. A 2 year-old child is receiving temporary total parental nutrition (TPN)


through a central venous line. This is the first day of TPN therapy. Although
all of the following nursing actions must be included in the plan of care of this
child, which one would be a priority at this time?
a. Use aseptic technique during dressing changes
b. Maintain central line catheter integrity
c. Monitor serum glucose levels
d. Check results of liver function tests

Ans. c. Monitor serum glucose levels

Rationale:
• Hyperglycemia may occur during the first day or 2 as the child adapts to the high-
glucose load of the TPN solution. Thus, a chief nursing responsibility is blood glucose
testing.

63. Mr. and Ms. Arora’s child failed to pass meconium within the first 24 hours
after birth; this may indicate which of the following?
a. Abdominal-wall defect
b. Celiac disease
c. Intussusception
d. Hirschsprung’s disease

Ans. d. Hirschsprung’s disease


34 National NORCET Test 6

Rationale:
• Failure to pass meconium within the first 24 hours after birth may be a sign of
Hirschsprung’s disease, a congenital anomaly resulting in mechanical obstruction due
to weak motility in an intestinal segment.
• Options A, B, & C: Failure to pass meconium is not connected with celiac disease,
intussusception, or abdominal-wall defect.

64. The given below image depicts:

a. Symmetric Tonic Neck Reflex


b. Asymmetric Tonic Neck Reflex
c. Palmar Mandibular Reflex
d. Palmer Mental Reflex

Ans. a. Symmetric Tonic Neck Reflex

Rationale:
Symmetric Tonic Neck Reflex Asymmetric Tonic Neck Reflex
• Limbs respond symmetrically. • Causes flexion on one side and extension on
• Its persistence may impede other motor the other.
milestones. • Not always seen in newborn.
• Onset: 35 weeks gestation. • Facilitates the development of bilateral body
• Fully developed: 1 month postnatal. awareness.
• Duration: 6-7 months postnatal.

65. Which of the following conditions most commonly causes acute


glomerulonephritis?
a. Prior infection with group A Streptococcus within the past 10-14 days
b. A congenital condition leading to renal dysfunction
c. Nephrotic syndrome
d. Viral infection of the glomeruli

Ans. a. Prior infection with group A Streptococcus within the past 10-14 days
National NORCET Test 6 35

Rationale:
• Acute glomerulonephritis is most commonly caused by the immune response to a
prior upper respiratory infection with group A Streptococcus.
• Post-Streptococcal Glomerulonephritis (PSGN) most commonly presents in children
1 to 2 weeks after a streptococcal throat infection, or within 6 weeks following a
streptococcal skin infection. Group A Streptococcus (GAS) has been subtyped
depending on the surface M protein and opacity factor, which are known to be
nephrogenic and can cause PSGN.
• Glomerulonephritis is not a congenital condition. PSGN is associated with skin
infections (impetigo) more frequently than throat infections (pharyngitis).
• Nephrotic syndrome is the combination of nephrotic-range proteinuria with a low
serum albumin level and edema. It is caused by increased permeability through
the damaged basement membrane in the renal glomerulus, especially infectious or
thrombo-embolic. It is the result of an abnormality of glomerular permeability that
may be primary with a disease-specific to the kidneys or secondary to congenital
infections, diabetes, systemic lupus erythematosus, neoplasia, or certain drug use.
• Glomerular lesions in acute GN are the result of glomerular deposition or in situ
formation of immune complexes. Poor hygiene, overcrowding, and low socioeconomic
status are important risk factors for streptococci outbreaks, and this explains the higher
incidence of PSGN in impoverished countries.

66. A nurse is assessing a newborn infant following circumcision and notes that
the circumcised area is red with a small amount of bloody drainage. Which of
the following nursing actions would be most appropriate?
a. Document the findings
b. Contact the physician
c. Circle the amount of bloody drainage on the dressing and reassess in 30 minutes
d. Reinforce the dressing

Ans. a. Document the findings. The penis is normally red during the healing process

Rationale:
• A yellow exudate may be noted in 24 hours, and this is a part of normal healing. The
nurse would expect that the area would be red with a small amount of bloody drainage.
• If the bleeding is excessive, the nurse would apply gentle pressure with sterile gauze.
If bleeding is not controlled, then the blood vessel may need to be ligated, and the
nurse would contact the physician. Because the findings identified in the question are
normal, the nurse would document the assessment.

67. The nurse answers a call bell and finds a frightened mother whose child, the
patient, is having a seizure. Which of these actions should the nurse take?
a. The nurse should insert a padded tongue blade in the patient’s mouth to prevent the
child from swallowing or choking on his tongue
b. The nurse should help the mother restrain the child to prevent him from injuring
himself
c. The nurse should call the operator to page for seizure assistance
d. The nurse should clear the area and position the client safely

Ans. d. The nurse should clear the area and position the client safely
36 National NORCET Test 6

Rationale:
• The primary role of the nurse when a patient has a seizure is to protect the patient from
harming him or herself.

68. Which of the following finding is normal in infants:-


a. Papilledema is rare in raised intracranial pressure
b. Elbow cross midline if passively done by examiner
c. Stroking patellar tendon of one side leads to contraction on opposite side
d. Parachute reflex
1. If a, b, c are correct
2. If a and c are correct
3. If b and d are correct
4. If all four (a, b, c, & d) are correct

Ans. 2. If a and c are correct


• Option (a): Papilloedema may be observed at almost any age, but it is relatively
uncommon in newborns and infants because of open fontanelles and raised intracranial
pressure manifests as sutural separation rather than as papilledema.
• Option (b): When done passively, elbow cross midline only in preterm babies.
• Option (c): “Stroking patellar tendon of one side leads to contraction on opposite side”
- This is called as crossed extensor reflex which is a normal occurrence in new born
babies.
• Option (d): Parachute reflex usually appears at around 9 months after birth and is
never present in newborn baby.

69. Projectile vomiting is a feature of:-


a. Esophageal atresia
b. Hypertrophic pyloric stenosis
c. Cholera
d. Intussusception

Ans. b. Hypertrophic pyloric stenosis

Rationale:
Esophageal atresia Frothing & bubbling at the mouth and nose after birth as well as
episodes of coughing, cyanosis, respiratory distress seen.
Hypertrophic pyloric Progressive non-bilious vomitingis the initial symptom of pyloric
stenosis stenosis.
Cholera Profuse watery diarrhea, which can progress to painless purging of
profuse rice water stools with a fishy smell is seen.
Intussusception Sudden onset of severe paroxysmal colicky pain that recurs at
frequent intervals & is accompanied by straining efforts with legs &
knees flexed. Vomiting occurs in most cases & is more frequent in the
early phase.
National NORCET Test 6 37

70. The image shows presence of:

a. Magenta tongue b. Black hairy tongue


c. Strawberry tongue d. Geographic tongue

Ans. c. Strawberry tongue

Rationale:
• Strawberry tongue/Raspberry tongue refers to glossitis which manifests with
hyperplastic (enlarged) fungiform papillae, giving the appearance of a strawberry.
• Strawberry tongue seen in Kawasaki disease.
• Magenta tongue: Purplish-red discoloration of the tongue, with edema and flattening
of the filiform papillae on the dorsum of the tongue. It occurs due to vitamin B2
(riboflavin) deficiency.

• Black hairy tongue: Black, hairy tongue is a harmless condition. It’s caused by a build-
up of dead skin cells on the tiny projections that contain taste buds (papillae). Potential
causes include broad-spectrum antibiotic and tobacco use.
38 National NORCET Test 6
• Geographic tongue: It is characterized by areas of atrophy and depapillation (loss
of papillae), leaving an erythematous (darker red) and smoother surface than the
unaffected areas. The depapillated areas are usually well, demarcated and bordered buy
as lightly raised, white, yellow or gray, serpiginous (snaking) peripheral zone.

MIDWIFERY & OBSTETRICAL NURSING


71. When assessing an aging adult woman, what does the nurse note as a normal
finding?
a. Rectocele b. Larger breasts
c. Vaginal dryness d. Severe osteoporosis

Ans. c. Vaginal dryness

Rationale:
• Vaginal dryness occurs with decreased estrogen and increased androgens circulating
in the aging female. This also leads to breast and genital atrophy, reduction in bone
mass, and increased atherosclerosis.
• A rectocele may occur and cause sexual or fecal elimination problems for the patient
that will need treatment.

72. The laboratory result of a specimen from a 20-year-old female patient shows
human papillomavirus (HPV). What would the nurse suspect the patient’s
diagnosis to be?
a. Syphilis b. Gonorrhea
c. Genital warts d. Genital herpes

Ans. c. Genital warts

Rationale:
• Genital warts are caused by human papillomavirus (HPV).
• Syphilis is caused by T. pallidum.
• Gonorrhea is caused by N. gonorrhoeae.
• Genital herpes are caused by herpes simplex virus.
National NORCET Test 6 39

73. When teaching a patient with problems of pelvic support to perform Kegel
exercises, what should the nurse tell the patient to do?
a. Contract her muscles as if trying to stop the flow of urine
b. Tighten the lower abdominal muscles over the bladder area
c. Squeeze all of the perineal muscles as if trying to close the vagina
d. Lie on the floor and do leg lifts to strengthen the abdominal muscles

Ans. a. Contract her muscles as if trying to stop the flow of urine

Rationale:
• The muscles that should be exercised are those affected by trying to stop a flow of urine.
• Kegel exercises help to strengthen muscular support of the perineum, pelvic floor, and
bladder and are also beneficial for problems with pelvic support and stress incontinence.

74. The patient’s diagnosis is a large rectocele requiring surgery. What nursing
interventions will be the priority postoperatively?
a. An ice pack to relieve swelling
b. An enema each day to relieve constipation
c. Administration of a stool softener each night
d. Perineal care after each urination or defecation

Ans. d. Perineal care after each urination or defecation

Rationale:
• The primary goal of care is to prevent wound infection and pressure on the vaginal
incision, which requires perineal cleansing at least twice daily and after each urination
and defecation.
• An ice pack and stool softener will be used but they are not the priority. The enema
would have been done preoperatively.

75. Identify the Type of Pelvic prolapse shown in the photograph:

a. Cystocoele b. Rectocoele
c. Enterocoele d. Uterine prolapse

Ans. d. Uterine prolapse


40 National NORCET Test 6

Rationale:

Uterine and Vaginal Prolapse

Fig: Types of uterine prolapse


National NORCET Test 6 41

Uterine prolapse:
Ĕ The severity of prolapse is indicated by increase in grade from I to IV.

Uterine Prolapse
Type Description
Grade I Cervix descends half way to the introitus.
Grade II Cervix descends to the introitus.
Grade III Cervix extends outside the introitus.
Grade IV The entire uterus, as well as the anterior and posterior vaginal walls, extends
or outside the introitus.
Procidentia

Vaginal Prolapse
Anterior Cystocele Herniation or
bulging of the
anterior vaginal
wall and overlying
bladder base into
the vaginal lumen.

Posterior Rectocele Herniation


or bulging of
the posterior
vaginal wall and
underlying rectum
into the vaginal
lumen.

Pouch of Enterocele Herniation of the


Douglas pouch of Douglas
containing small
bowel into the
vaginal lumen.

76. Which diagnostic tests of the female reproductive system are operative
procedures requiring surgical anesthesia?
a. D&C b. Conization
c. Culdoscopy d. Endometrial biopsy
1. If a, b, c are correct 2. If a and c are correct
3. If b and d are correct 4. If all four (a, b, c, & d) are correct

Ans. 1. If a, b, c are correct


42 National NORCET Test 6

Rationale:
• A Culdoscopy involves insertion of an endoscope through an incision made through
the posterior fornix of the cul-de-sac and requires surgical anesthesia, as does the
removal of cervical tissue during a conization.
• A D&C and laparoscopy are also operative procedures requiring surgical anesthesia.
• Colposcopy and endometrial biopsies do not require surgical anesthesia.

77. Although an 18-year-old girl knows that abstinence is one way to prevent STIs,
she does not consider that as an alternative. She asks the nurse at the clinic if
there are other measures for preventing STIs. What should the nurse teach her?
a. Abstinence is the only way to prevent STIs.
b. Voiding immediately after intercourse will decrease the risk for infection.
c. A vaccine can prevent genital warts and cervical cancer caused by some strains of HPV.
d. Thorough hand washing after contact with genitals can prevent oral-genital spread of
STIs.

Ans. c. A vaccine can prevent genital warts and cervical cancer caused by some strains of HPV

Rationale:
• A vaccine is available for HPV types 6, 11, 16, and 18 that protects against genital warts
and cervical cancer.
• Although sexual abstinence is the most certain method of avoiding all STIs, it is not
usually a feasible alternative.
• Undamaged condoms also serve to protect against infection.
• Conscientious hand washing and voiding after intercourse are positive hygienic
measures that will help to prevent secondary infections but will not prevent STIs.

78. A 29-year-old woman is at the clinic with menorrhagia. What finding would
the nurse expect in this patient’s assessment?
a. Pain with each menstrual period
b. Excessive bleeding at irregular intervals
c. Bleeding or spotting between menstrual periods
d. Increased duration or amount of menstrual bleeding

Ans. d. Increased duration or amount of menstrual bleeding

Rationale:
• Menorrhagia is increased duration or amount of bleeding with menses.
• Pain with menses is called dysmenorrhea.
• Metrorrhagia is excessive bleeding at irregular intervals or spotting between menstrual
periods.
National NORCET Test 6 43

79. A patient with abdominal pain and irregular vaginal bleeding is admitted to
the hospital with a suspected ectopic pregnancy. Before actual diagnosis, what
is the most appropriate action by the nurse?
a. Provide analgesics for pain relief
b. Monitor vital signs, pain, and bleeding frequently
c. Explain the need for frequent blood samples for β-human chorionic gonadotropin
monitoring
d. Offer support for the patient’s emotional response to the loss of the pregnancy

Ans. b. Monitor vital signs, pain, and bleeding frequently

Rationale:
• Ectopic pregnancy is a life-threatening condition. If the fallopian tube ruptures, profuse
bleeding can lead to hypovolemic shock. All of the interventions are indicated but the
priority is monitoring the vital signs and pain for evidence of bleeding.

80. Which among the following is the most common side effect of contraceptive
shown in the photograph below?

a. Pain
b. PID
c. Bleeding
d. Perforation

Ans. c. Bleeding

Rationale:

Complications of IUCD
• Most common complication: Bleeding (Increased blood loss of 80mL/cycle).
• 2nd most common: Pain (Most common reason for removal of IUCD).
• Expulsion (patients should be encouraged to look/feel for string monthly rate of CuT):
8-10%.
• Infection: Most typical infection associated with Cu T use is Actinomyces (Seen in
8-20% cases).
44 National NORCET Test 6

Clinical Pearls:
If a woman develops a gonococcal or chlamydial infection, she is much more likely to develop PID
if she has an IUD in place.
• PID development risk is greatest in the first 20 days after IUD insertion and rare
thereafter.
• Uterine perforation (Most common time for perforation is at the time of insertion).
• Relationship between IUD use and ectopic pregnancy: Women using an IUD are
much less likely to conceive compared to women not using any form of birth control.
However, in the unlikely event that they do conceive, there is a greater probability that
it will be ectopic.

IUD associated with Side-effects / Complications


Side-effects / Complications IUD Most Commonly associated
Highest Pregnancy rate Lippes Loop
Lowest Pregnancy Rate LNH-IUD
Highest expiulsion Rate Lippes Loop
Lowest expiulsion Rate Progestasert
Highest Removal Rate LNH-IUD
Lowest Removal Rate Progestasert

81. What is the fertility test that requires a couple to have sexual intercourse at the
time of ovulation and come for testing 2 to 8 hours after intercourse?
a. Huhner’s test b. Semen analysis
c. Endometrial biopsy d. Hysterosalpingogram

Ans. a. Huhner’s test

Rationale:
• Huhner’s (or Sims-Huhner) test involves examination of a mucus sample of the cervix
within 2 to 8 hours after intercourse to determine the number and mobility of sperm
in the cervical mucus.
• A semen analysis is a simple examination of semen for the number, mobility, and
structure of sperm.
• An endometrial biopsy provides a sample of endometrium to evaluate its changes
under the influence of progesterone.
• A hysterosalpingogram is a contrast x-ray of the uterine cavity and fallopian tubes.

82. An infertile couple has used at-home ovulation testing using basal body
temperature without conceiving. The nurse understands that what will be used
first to treat this infertile couple?
a. Surgery to reduce endometriosis
b. Intrauterine insemination with sperm from the husband
c. Selective estrogen receptor modulator (clomiphene)
d. Assisted reproductive technologies (e.g., in vitro fertilization [IVF])

Ans. c. Selective estrogen receptor modulator clomiphene


National NORCET Test 6 45

Rationale:
• Drug therapy will be used before more invasive treatments. Drugs may include selective
estrogen receptor modulators, menotropin (human menopausal gonadotropin),
follicle-stimulating hormone agonists, gonadotropin-releasing hormone (GnRH)
antagonists, GnRH agonists, or human chorionic gonadotropin (hCG).
• If the husband’s reproductive system is functioning, intrauterine insemination with his
sperm will be done.
• The assisted reproductive technologies may be used if this is not successful.
• The surgery for endometriosis could be done if this was diagnosed but that is not
included in this question.

83. What should the nurse include when implementing care for the patient with
acute PID?
a. Perform vaginal irrigations every 4 hours
b. Promote bed rest in semi-Fowler’s position
c. Instruct the patient to use tampons to control vaginal drainage
d. Ambulate the patient frequently to promote drainage of exudate

Ans. b. Promote bed rest in semi-Fowler’s position

Rationale:
• Bed rest in semi-Fowler’s position promotes drainage of the pelvic cavity by gravity and
may prevent the development of abscesses high in the abdomen.
• Coitus, douching, and tampon use should be avoided to prevent spreading infection
upward from the vagina, although frequent perineal care should be performed to
remove infectious drainage.

84. A patient with a 10-week pregnancy is admitted to the emergency department


with vaginal bleeding and abdominal cramping. What does the nurse recognize
about this situation?
a. The patient will recover quickly when the bleeding stops
b. The patient is most likely experiencing a spontaneous abortion
c. The patient will be scheduled for an immediate dilation and curettage (D&C)
d. Treatment of the patient with bed rest is usually successful in preventing further
bleeding

Ans. b. The patient is most likely experiencing a spontaneous abortion

Rationale:
• In the presence of a confirmed pregnancy, uterine cramping with vaginal bleeding is
the most important sign of spontaneous abortion.
• Other conditions causing vaginal bleeding, such as an incompetent cervix, do not
usually cause cramping. There is no evidence that any medical treatment improves the
outcome for spontaneous abortion.
• Blood loss can be significant and the loss of the pregnancy may cause long-term
grieving.
• Dilation and curettage (D&C) (if needed) is performed after the abortion to minimize
blood loss and reduce the chance of infection.
46 National NORCET Test 6

85. Identify the Type of placenta insertion shown in image:-

a. Velamentous
b. Circummarginate
c. Succenturiate
d. Circumvallate

Ans. b. Circummarginate

Rationale:
Circummarginate placenta Vs Circumvallate placenta

Circummarginate placenta Circumvallate placenta


A flattened edge with a ridge of fibrin A peripheral cuplike insertion of the
demarking the edge of the vascular plate. membranes at the placental surface.
It often only involves a portion of the
circumference.

Placenta Succenturiata

• When one or more accessory lobes are situated away from the main placenta, but the
two portions connected by the membranes through which blood vessels run are called
Placenta Succenturiata.
National NORCET Test 6 47

• How does it develop?


Ĕ Ans: When a part of chorionic leave is well developed (normally degenerate), it
forms accessory lobes of Placenta Succenturiata.
• How you diagnose Placenta Succenturiata?
Ĕ Ans: A small lobe attached with the membranes through which blood vessels run
from small lobes to the main placenta. In case of missing lobes there is a hole in the
membranes with umbilical vessels running upto the margin of the hole.
• What is the clinical importance of the Placenta Succenturiata?
Ĕ Ans: There is a chance of retained lobe of placenta which may cause PPH, Infection,
and sub-involution.

Velamentous insertion Vs Marginal insertion of Placenta


Velamentous insertion Marginal insertion

86. Following a dilation and curettage (D&C), what complication is most important
for the nurse to assess the patient for?
a. Infection b. Hemorrhage
c. Urinary retention d. Perforation of the bladder

Ans. b. Hemorrhage

Rationale:
• The risk for bleeding is increased following a dilation and curettage (D&C) because the
endometrial lining is scraped and injury to the uterus can occur.
• The nurse should closely assess the amount of bleeding with frequent pad checks for
the first 24 hours. Infection following D & C is uncommon and the urinary system is
not affected.

87. The patient is a perimenopausal woman who has an unexpected and unwanted
pregnancy. She wants an abortion. What should the nurse teach her about the
effects of an abortion?
a. D & C will be needed
b. She will feel much better afterwards
c. The products of conception will pass immediately
d. She will need someone to support her through her loss

Ans. d. She will need someone to support her through her loss
48 National NORCET Test 6

Rationale:
• There is physical and emotional pain and grieving after an abortion that puts the
patient in need of support.
• D & C is needed only if the products of conception do not pass completely or bleeding
becomes excessive.
• The time it takes for the products of conception to pass depends on the type of abortion
being done and is immediate with surgical abortion and slower with medical abortion.

88. Premenstrual syndrome (PMS) is most likely to be diagnosed in a woman with


which occurrence?
a. Symptoms can be controlled with the use of progesterone
b. The woman has symptoms only when oral contraceptives are used
c. Symptoms can be correlated with altered serum levels of estrogen and progesterone
d. The woman has the same symptom pattern following ovulation for two or three
consecutive menstrual cycles

Ans. d. The woman has the same symptom pattern following ovulation for two or three
consecutive menstrual cycles

Rationale:
• Premenstrual syndrome (PMS) is diagnosed when other possible causes for symptoms
have been eliminated.
• A diagnosis is based on a symptom diary that indicates the same symptoms during
the luteal phase, approximately 1 week before menses, for two or three consecutive
menstrual cycles.
• Oral contraceptives may be used to control the symptoms of PMS by suppressing
ovulation and although progesterone may also relieve the symptoms of PMS, its
effectiveness is not associated with the diagnosis of PMS.
• There are no laboratory findings that account for the premenstrual symptoms.

89. A 25-year-old female patient is at the clinic and says that she has white vaginal
drainage and itching. Which etiology would be suspected?
a. Cancer
b. Candidiasis
c. Trichomonas vaginalis
d. Bacterial vaginosis infection

Ans. b. Candidiasis

Rationale:
• Candidiasis is a white, thick or curdy discharge and causes itching and inflammation.
• Cancer would be more likely to produce a bloody discharge.
• Trichomonas vaginalis produces a malodorous frothy green or yellow discharge.
• Bacterial vaginosis infection produces copious amounts of thin gray or white drainage
with a fishy odor.
National NORCET Test 6 49

90. Clinical demonstration of fetal heart rate shown in the photograph is known
as:-

a. Pinard stethoscope
b. Bladder Sound
c. Doppler USG
d. Cusco’s speculum

Ans. a. Pinard stethoscope

Rationale:
• A Pinard horn is a type of stethoscope used to listen to the heart rate of a fetus during
pregnancy. It is a hollow horn, often made of wood or metal, about 8 inches (200 mm)
long. It functions similarly to an ear trumpet by amplifying sound.

Cusco’s bivalved self-retaining vaginal speculum


• Material: Stainless steel.
• Sterilization: Autoclaving and boiling.

• Uses:
Ĕ Routine per speculum examination in gynecology
Ĕ Colposcopy
Ĕ Endometrial biopsy
Ĕ Cervical punch biopsy
Ĕ Pap smear
Ĕ Insertion and removal of intrauterine contraceptive device (IUCO)
Ĕ Intrauterine insemination (IUI).

Bladder Sound
• Material: Stainless steel
• Sterilization: Autoclaving and boiling.
50 National NORCET Test 6

• Uses:
Ĕ To define the limits of bladder during operations.
Ĕ To confirm a suspected bladder injury during vaginal hysterectomy.
Ĕ To determine length and direction of VVF.
Ĕ To sound a calculus or foreign body in the bladder.
Ĕ To differentiate bladder or urethral diverticulum from anterior vaginal wall cyst.

91. The patient is suspected of having endometriosis and/or uterine leiomyoma.


What best describes what is found with these conditions?
a. Endometriosis and uterine leiomyoma are two gynecologic conditions that increase
with the onset of menopause
b. Danazol (Danacrine) and Lupron (GnRH analog) are used to treat endometriosis and
leiomyomas to create a pseudopregnancy
c. Treatment of endometriosis and leiomyomas depends on the severity of symptoms and
the woman’s desire to maintain fertility
d. The presence of ectopic uterine tissue that bleeds and causes pelvic and abdominal
adhesions, cysts, and pain is known as uterine leiomyoma

Ans. c. Treatment of endometriosis and leiomyomas depends on the severity of symptoms and
the woman’s desire to maintain fertility

Rationale:
• The treatment of endometriosis and leiomyomas is surgical when the patient does not
tolerate the symptoms, with the type of surgery for endometriosis dependent on the
desire for pregnancy.
• Endometriosis and leiomyomas subside with the onset of menopause. Therefore, the
medications to treat them create a pseudomenopause.
• The ectopic uterine tissue is endometriosis, while leiomyomas are fibrous smooth
muscle tumors.

92. An 18-year-old patient with irregular menstrual periods, hirsutism, and


obesity has been diagnosed with polycystic ovary syndrome (PCOS). What is
an accurate rationale for the expected treatment?
a. Hirsutism may be treated with leuprolide to decrease an altered body image
b. The medication used will cure the hormonal abnormality of excess testosterone
c. The loss of weight will improve all of the symptoms, so this will be the first treatment
tried
d. The progression of PCOS leads to cardiovascular disease and possibly to type 2 diabetes
mellitus if untreated

Ans. d. The progression of PCOS leads to cardiovascular disease and possibly to type 2 diabetes
mellitus if untreated
National NORCET Test 6 51

Rationale:
• Left untreated, polycystic ovary syndrome (PCOS) may lead to cardiovascular disease
and abnormal insulin resistance with type 2 diabetes mellitus. Hirsutism may be
treated with spironolactone.
• Leuprolide is used to treat hyperandrogenism but PCOS cannot be cured.
• Severity of symptoms is associated with obesity but the hormone abnormalities will
be treated along with the obesity to prevent complications. If this treatment is not
successful, a hysterectomy with bilateral salpingectomy and oophorectomy may be
performed.

93. A patient with a stage 0 cervical cancer identified from a Papanicolaou (Pap)
test asks the nurse what this finding means. The nurse’s response should include
which information?
a. Malignant cells have extended beyond the cervix to the upper vagina
b. Abnormal cells are present but are confined to the epithelial layer of the cervix
c. Atypical cells characteristic of inflammation but not necessarily malignancy are present
d. This is a common finding on Pap testing and she will be examined frequently to see
whether the abnormal cells spread beyond the cervix

Ans. b. Abnormal cells are present but are confined to the epithelial layer of the cervix

Rationale:
• A stage 0 cervical cancer indicates cancer in situ that is confined to the epithelial layer
of the cervix and requires treatment.
• Stage 0 is the least invasive. Stage I is confined to the cervix.
• Stage II has spread beyond the cervix to the upper two thirds of the vagina but not the
tissues around the uterus.
• Stage III involves the pelvic wall, lower third of the vagina, and/or kidney problems.
• Stage IV indicates spread to distant organs.

94. A patient has been diagnosed with cancer of the ovary. In planning care for the
patient, what does the nurse recognize that treatment of the patient depends
on?
a. Results of a direct-needle biopsy of the ovary
b. Results of a laparoscopy with multiple biopsies
c. Whether the patient desires to maintain fertility
d. The findings of metastasis by ultrasound or computed tomography (CT) scan

Ans. b. Results of a laparoscopy with multiple biopsies

Rationale:
• Treatment of ovarian cancer is determined by staging from the results of laparoscopy
with multiple biopsies of the ovaries and other tissue throughout the pelvis and lower
abdomen.
• The patient’s desire for fertility is not a consideration because of the high mortality rate
associated with ovarian cancer.
52 National NORCET Test 6
• Although diagnosis of ovarian tumors may be made by transvaginal ultrasound or
computed tomography (CT) scan, the treatment of ovarian cancer depends on the
staging of the tumor.

95. Identify the placental condition shown in the picture below?

a. Placenta previa b. Placenta accreta


c. Placenta percreta d. Placenta increta

Ans. a. Placenta previa

Rationale:

Types of placenta Previa


National NORCET Test 6 53

• Total/Complete/Central previa: It is found when the placenta completely covers the


internal os of the cervix. This is the most dangerous location because of its potential for
hemorrhage. Most commonly seen in approximately 30% of all previas.
• Partial previa: It exists when the placenta partially covers the internal os.
• Marginal previa: Implantation in the lower uterine segment is such that the placental
edge does not cover the internal os but lies > 2-cm wide perimeter around the os.
• Low-lying previa: Implantation in the lower uterine segment is such that the placental
edge does not cover the internal os but lies within a 2 cm wide perimeter around the
os. It has no significance.

Placenta accreta Vs Placenta increta Vs Placenta percreta


Placenta accreta: Placenta villi adhere to
myometrium without an intervening layer of
decidua.
Placenta increta: Villi within the myometrium
usually involving previous cesarean section.
Placenta percreta: Villi penetrate through the
uterine wall to the serosa.

MEDICAL SURGICAL NURSING


96. Mr. Pathak has started a new drug for hypertension. Thirty minutes after he
takes the drug, he develops chest tightness and becomes short of breath and
tachypnea. He has a decreased level of consciousness. These signs indicate
which of the following conditions?
a. Pulmonary embolism b. Asthma attack
c. Respiratory failure d. Rheumatoid arthritis

Ans. c. Respiratory failure

Rationale:
• The client was reacting to the drug with respiratory signs of impending anaphylaxis,
which could eventually lead to respiratory failure.
• Although the signs are also related to a pulmonary embolism, consider the new drug
first.
• The symptoms may look like an asthma attack, but it may change because of the new
drug ingested.
• Rheumatoid arthritis is an autoimmune and inflammatory disease, which means that
the immune system attacks healthy cells in the body by mistake, causing inflammation
in the affected parts of the body.
54 National NORCET Test 6

97. Which patient should you, as charge nurse, assign to a new graduate Registered
Nurse (RN), who is orienting to the neurologic unit?
a. An 85-year-old dementia patient to be transferred to long-term care today
b. A 28-year-old newly admitted patient with spinal cord injury
c. A 54-year-old patient with Parkinson’s who needs assistance with bathing
d. A 67-year-old patient with stroke 3 days ago and left-sided weakness

Ans. d. A 67-year-old patient with stroke 3 days ago and left-sided weakness

Rationale:
• The new graduate RN who is oriented to the unit should be assigned stable, non-
complex patients, such as the patient with stroke.
• Option A: A patient for transfer should be assigned to a nurse who has experience in
the process of transferring patients.
• Option B: The newly admitted SCI should be assigned to experienced nurses. Most
cases of SCI take place when trauma breaks and squeezes the vertebrae, or the bones
of the back. This, in turn, damages the axons—the long nerve cell “wires” that pass
through vertebrae, carrying signals between the brain and the rest of the body. The
axons might be crushed or completely severed by this damage. Someone with injury to
only a few axons might be able to recover completely from their injury. On the other
hand, a person with damage to all axons will most likely be paralyzed in the areas
below the injury.
• Option C: The patient with Parkinson’s disease needs assistance with bathing, which is
best delegated to the nursing assistant.

98. Rohon falls off his bicycle and injures his ankle. Which of the following actions
shows the initial response to the injury in the extrinsic pathway?
a. Release of tissue thromboplastin
b. Conversion of factor VIII to factor VIIIa
c. Release of Calcium
d. Conversion of factors XII to factor XIIa

Ans. a. Release of tissue thromboplastin

Rationale:
• The coagulation pathway is a cascade of events that leads to hemostasis. The intricate
pathway allows for rapid healing and prevention of spontaneous bleeding. Two paths,
intrinsic and extrinsic, originate separately but converge at a specific point, leading to
fibrin activation. The purpose is to ultimately stabilize the platelet plug with a fibrin
mesh.
• The intrinsic pathway is activated through exposed endothelial collagen, and the
extrinsic pathway is activated through tissue factor released by endothelial cells after
external damage.
• Tissue thromboplastin is released when damaged tissue comes in contact with clotting
factors.
• Intrinsic Pathway:
Ĕ This pathway is the longer pathway of secondary hemostasis.
National NORCET Test 6 55

Ĕ It begins with the activation of Factor XII (a zymogen, inactivated serine protease)
which becomes Factor XIIa (activated serine protease) after exposure to endothelial
collagen. Endothelial collagen is only exposed when endothelial damage occurs.
Ĕ Factor XIIa acts as a catalyst to activate factor XI to Factor XIa. Factor Xia then
goes on to activate factor IX to factor IXa. Factor IXa goes on to serve as a catalyst
for turning factor X into factor Xa. This is known as a cascade.
Ĕ When each factor is activated, it goes on to activate many more factors in the next
steps. As you move further down the cascade, the concentration of that factor
increases in the blood.
Ĕ For example, the concentration of factor IX is more than that of factor XI. When
factor II is activated by either intrinsic or extrinsic pathway, it can reinforce the
intrinsic pathway by giving positive feedback to factors V, VII, VIII, XI, and XIII.
This makes factor XII less critical; patients can actually clot well without factor XII.
The intrinsic pathway is clinically measured as the partial thromboplastin time
(PTT).
• Extrinsic Pathway:
Ĕ The extrinsic pathway is the shorter pathway of secondary hemostasis.
Ĕ Once the damage to the vessel is done, the endothelial cells release tissue factor
which goes on to activate factor VII to factor VIIa.
Ĕ Factor VIIa goes on to activate factor X into factor Xa. This is the point where both
extrinsic and intrinsic pathways become one. The extrinsic pathway is clinically
measured as the prothrombin time (PT).
• Calcium is released to assist the conversion of factors X to Xa.
• Conversions of factors XII to XIIa are part of the intrinsic pathway.
• Conversion factors VIII to VIIIa are part of the intrinsic pathway.

99. Which roommate would be most suitable for the 6-year-old male with a
fractured femur in Russell’s traction?
a. 6-year-old male with osteomyelitis
b. 12-year-old male with a fractured femur
c. 16-year-old female with scoliosis
d. 10-year-old male with sarcoma

Ans. b. 12-year-old male with a fractured femur

Rationale:
• The 6-year-old should have a roommate as close to the same age as possible, so the
12-year-old is the best match. A bed is available and the patient gets assigned. There are
certain constraints—sex, semi-private versus private, isolation issues, acuity, telemetry
and specialty needs. All need to be taken into account to ensure that each patient
goes to the right place and receives the proper care. But good capacity management
demands that bed assignment be carefully considered and executed.
• The most suitable patient is 12 years old patient with fracture of femur as this patient
has similar diagnosis with the patient present in our scenario.
• Option A: The 6-year-old with osteomyelitis is infectious. Rooms and beds belong to
the critical assets of just any hospital.
56 National NORCET Test 6
• Option C: The client is too old and is female. Bed assignment simply provides the
proper location based on specific patient attributes like sex, isolation, telemetry, acuity,
and specialty needs.
• Option D: The 10-year-old with sarcoma has cancer and will be treated with
chemotherapy that makes him immune suppressed. Bed managers aim at finding an
assignment of patients to rooms that strikes a balance between patients’ preferences
and comfort on the one hand, and patients’ clinical conditions and the resulting
required room facilities on the other.

100. The malformation is shown below?

a. Omphalocele b. Gastroschisis
c. Icthyosis d. Bochdalek hernia

Ans. b. Gastroschisis

Rationale:
• Gastroschisis is a birth defect of the abdominal (belly) wall. The baby’s intestines are
found outside of the baby’s body, exiting through a hole beside the belly button. The
hole can be small or large and sometimes other organs, such as the stomach and liver,
can also be found outside the baby’s body.
• Notice that the image shows the abdominal wall defect which is located at the junction
of the umbilicus and normal skin. The defect occurs 5-8 weeks after conception, most
likely due to a disruption of the blood supply to the developing abdominal wall. Since
there is no overlying sac or peritoneum the diagnosis is Gastroschisis.
• Omphalocele is another congenital birth defect, but it involves the umbilical cord
itself, and the organs remain enclosed in visceral peritoneum. With omphalocele, the
defect is usually much larger than the Gastroschisis.
• Bochdalek hernia: It is a form of congenital diaphragmatic hernia. They occur
posteriorly and are due to a defect in the posterior attachment of the diaphragm when
there is a failure of pleuroperitoneal membrane closure in utero.
• Ichthyosis is a family of genetic skin disorders characterized by dry, thickened, scaly
skin.
National NORCET Test 6 57

101. Mr. Jaydeep develops hepatic encephalopathy. Which clinical manifestation is


most common with this condition?
a. Increased urine output
b. Decreased tendon reflex
c. Altered level of consciousness
d. Hypotension

Ans. c. Altered level of consciousness

Rationale:
• Hepatic encephalopathy is defined as a spectrum of neuropsychiatric abnormalities in
patients with liver dysfunction, after exclusion of brain disease. Hepatic encephalopathy
is characterized by personality changes, intellectual impairment, and a depressed level
of consciousness.
• Hepatic encephalopathy is caused when toxins that are normally cleared from the
body by the liver accumulate in the blood, eventually travelling to the brain. Many of
the symptoms of hepatic encephalopathy are reversible when promptly detected and
treated.
• Changes in behavior and level of consciousness are the first signs of hepatic
encephalopathy. Hepatic encephalopathy is caused by liver failure and develops when
the liver is unable to convert protein metabolic product ammonia to urea. This results
in accumulation of ammonia and other toxics in the blood that damages the cells.
• The main cause of renal dysfunction in in-patients with liver disease is prerenal failure;
specifically, the cause is a reduction in perfusion of the kidneys.
• It must be emphasized that the flapping tremor of the extremities is also observed in
patients with uremia, pulmonary insufficiency, and barbiturate toxicity.
• A diseased liver can cause portal hypertension, which is high blood pressure in the
portal vein. The portal vein supplies the liver with blood. Over time, this pressure causes
blood vessels to grow, called collateral blood vessels. These vessels act as channels to
divert the blood under high pressure.

102. Which action by the nurse indicates understanding of herpes zoster?


a. The nurse administers a prescribed antibiotic
b. The nurse covers the lesions with a sterile dressing
c. The nurse administers oxygen
d. The nurse wears gloves when providing care

Ans. d. The nurse wears gloves when providing care

Rationale:
• Shingles is also known as Herpes zoster/zoster.. Clients with shingles should be placed
in contact precautions. Wearing gloves during care will prevent transmission of the
virus. Use universal precautions in caring for the client to prevent transmission of
disease to self or other clients. VZV can be transmitted to others and cause chickenpox
in the person who has not previously had the disease.
• Antibiotics are not prescribed for herpes zoster. Instruct the client in the use of antiviral
medications, as prescribed.
58 National NORCET Test 6
• Covering the lesions with a sterile gauze is not necessary. Suggest the use of gauze
to separate the lesions in skin folds. This reduces irritation, itching, and cross-
contamination.
• Oxygen is not necessary for shingles. Instruct the client to avoid contact with pregnant
women and immunocompromised individuals.

103. A client is admitted for an MRI. The nurse should question the client regarding:
a. A titanium hip replacement
b. Inability to move his feet
c. Pregnancy
d. Allergies to antibiotics

Ans. c. Pregnancy

Rationale:
• Although there is no evidence to suggest MRI scans can pose a risk during pregnancy,
it is considered as a precaution to not perform MRI during pregnancy, particularly
in the first three months. This is particularly the case during the first trimester of
pregnancy, as organogenesis takes place during this period. The concerns in pregnancy
are the same as for MRI in general, but the fetus may be more sensitive to the effects—
particularly to heating and to noise.
• Clients with a titanium hip replacement can have an MRI. Some other devices and
implants might be contraindicated.
• The client should remain still only when instructed. Patients who are unable to be still
or obey breathing instructions in the scanner need special attention.
• No antibiotics are used with this test. MRI contrast agents are gadolinium chelates
with different stability, viscosity, and osmolality. Gadolinium is a relatively very safe
contrast; however, it rarely might cause allergic reactions in patients.

104. The nursing assistant reports to you, the RN, that the patient with myasthenia
gravis (MG) has an elevated temperature (102.20 F), heart rate of 120/minute,
rise in blood pressure (158/94), and was incontinent of urine and stool. What is
your best first action at this time?
a. Recheck vital signs in 1 hour
b. Administer an acetaminophen suppository
c. Notify the physician immediately
d. Reschedule patient’s physical therapy session

Ans. c. Notify the physician immediately

Rationale:
• Myasthenia gravis is a chronic autoimmune, neuromuscular disease that causes
weakness in the skeletal muscles that worsens after periods of activity and improves
after periods of rest.
• Myasthenia gravis is caused by an abnormal immune reaction (antibody-mediated
autoimmune response) in which the body’s immune defenses (i.e., antibodies)
inappropriately attack certain proteins in muscles that receive nerve impulses.
National NORCET Test 6 59

• The changes that the nursing assistant is reporting are characteristics of myasthenia
crisis, which often follows some type of infection. The patient is at risk for inadequate
respiratory function. In addition to notifying the physician, the nurse should carefully
monitor the patient’s respiratory status. The patient may need intubation and
mechanical ventilation.
• Option A: This patient’s vital signs need to be re-checked sooner than 1 hour.
• Option B: The nurse would notify the physician before giving the suppository because
there may be orders for cultures before giving acetaminophen.
• Option D: Rescheduling the physical therapy can be delegated to the unit clerk and is
not urgent. Focus: Prioritization.

105. A nurse is viewing the cardiac monitor in a client’s room and notes that the
client has just gone into ventricular tachycardia shown in the image. The client
is awake and alert and has good skin color. The nurse would prepare to do
which of the following?

a. Immediately defibrillate
b. Prepare for pacemaker insertion
c. Administer amiodarone intravenously
d. Administer epinephrine (Adrenaline) intravenously

Ans. c. Administer amiodarone intravenously

Rationale:
• Ventricular tachycardia (VT): A life-threatening dysrhythmia that originates from an
irritable focus within the ventricle at a rapid rate. Because the ventricles are capable of
an inherent rate of 40 beats/min or less, a ventricular rhythm at a rate of 100 beats/min
may be considered tachycardia.
• First-line treatment of ventricular tachycardia in a client who is hemodynamically
stable is the use of anti-dysrhythmics such as amiodarone (Cordarone), lidocaine
(Xylocaine), and procainamide (Pronestyl).
• Cardioversion also may be needed to correct the rhythm (cardioversion is recommended
for stable ventricular tachycardia).
• Defibrillation is used with pulseless ventricular tachycardia.
• Epinephrine would stimulate already excitable ventricle and is contraindicated.
60 National NORCET Test 6

106. The nurse is aware that the following terms used to describe reduced cardiac
output and perfusion impairment due to ineffective pumping of the heart is:
a. Anaphylactic shock b. Cardiogenic shock
c. Distributive shock d. Myocardial infarction (MI)

Ans. b. Cardiogenic shock

Rationale:
• Cardiogenic shock is shock related to ineffective pumping of the heart.
• Anaphylactic shock results from an allergic reaction. This severe reaction happens
when an over-release of chemicals puts the person into shock.
• Distributive shock results from changes in the intravascular volume distribution and is
usually associated with increased cardiac output.
• MI isn’t a shock state, though in most cases, a lack of oxygen to the heart, usually
from a heart attack, damages its main pumping chamber. Without oxygen-rich
blood circulating to that area of the heart, the heart muscle can weaken and go into
cardiogenic shock.

107. A patient is admitted to the hospital with a diagnosis of primary


hyperparathyroidism. A nurse checking the patient’s lab results would expect
which of the following changes in laboratory findings?
a. Low urine calcium
b. Elevated serum calcium
c. Elevated serum vitamin D
d. Low serum parathyroid hormone (PTH)

Ans. b. Elevated serum calcium

Rationale:
• The parathyroid glands regulate the calcium level in the blood. In hyperparathyroidism,
the serum calcium level will be elevated.
• A normal PTH in the presence of hypercalcemia is considered inappropriate and still
consistent with PTH-dependent hypercalcemia. PTH levels should be very low in
those patients with PTH-independent hypercalcemia.
• A comprehensive clinical evaluation complemented by routine laboratory and radiologic
studies should be sufficient to establish a diagnosis of primary hyperparathyroidism in
a patient with persistent hypercalcemia and an elevated serum level of parathyroid
hormone.
• Urine calcium may be elevated, with calcium spilling over from elevated serum levels.
This may cause renal stones.
• Parathyroid hormone levels may be high or normal but not low. The body will lower
the level of vitamin D in an attempt to lower calcium.
• Parathyroid hormone levels may be high or normal but not low. Patients with primary
hyperparathyroidism and other causes of PTH-dependent hypercalcemia often have
frankly elevated levels of PTH, while some will have values that fall within the reference
range for the general population.
National NORCET Test 6 61

108. A nurse is performing a routine assessment of an IV site in a patient receiving


both IV fluids and medications through the line. Which of the following would
indicate the need for discontinuation of the IV line as the next nursing action?
a. The IV solution is infusing too slowly, particularly when the limb is elevated
b. The patient complains of pain from movement
c. A hematoma is visible in the area of the IV insertion site
d. The area proximal to the insertion site is reddened, warm, and painful

Ans. d. The area proximal to the insertion site is reddened, warm, and painful

Rationale:
• An IV site that is red, warm, painful and swollen indicates that phlebitis has developed
and the line should be discontinued and restarted at another site.
• Phlebitis is inflammation of a vein. It is usually associated with acidic or alkaline
solutions or solutions that have a high osmolarity. Phlebitis can also occur as a result of
vein trauma during insertion, use of an inappropriate I.V. catheter size for the vein, or
prolonged use of the same I.V. site.
• An IV line that is running slowly may simply need flushing or repositioning. Monitor
administration rates and inspect the I.V. site frequently. Change the infusion site
according to the facility’s policy.
• Pain on movement should be managed by maneuvers such as splinting the limb with
an IV board or gently shifting the position of the catheter before making a decision to
remove the line. Apply warm, moist compresses to the area. Document the patient’s
condition and interventions.
• A hematoma at the site is likely a result of minor bleeding at the time of insertion and
does not require discontinuation of the line.

109. A patient with a spinal cord injury at level C3-4 is being cared in the ED. What
is the priority assessment?
a. Assess the level at which the patient has retained mobility
b. Monitor respiratory effort and oxygen saturation level
c. Determine the level at which the patient has intact sensation
d. Check blood pressure and pulse for signs of spinal shock

Ans. b. Monitor respiratory effort and oxygen saturation level

Rationale:
• A spinal cord injury (SCI) is damage to the spinal cord that results in a loss of function,
such as mobility and/or feeling. Frequent causes of spinal cord injuries are trauma (car
accident, gunshot, falls, etc.) or disease (polio, spina bifida, Friedreich’s ataxia, etc.).
• The first priority for the patient with an SCI is assessing respiratory patterns and
ensuring an adequate airway. The patient with a high cervical injury is at risk for
respiratory compromise because the spinal nerves (C3 – 5) innervate the phrenic
nerve, which controls the diaphragm.
• Option A: This data can be assessed after monitoring the respiratory effort and oxygen
saturation level of the patient.
• Option C: Determination of this data can be done after addressing the concerns on the
respiratory status of the patient.
62 National NORCET Test 6
• Option D: Vital signs checking is also necessary, but not a high priority. Vital signs
can be quite abnormal following SCI. In addition to the usual causes in trauma such
as pain, bleeding, and distress, this can be due to loss of autonomic control, which
occurs particularly in cervical or high thoracic injuries. The autonomic nervous system
controls our HR, BP, temperature, etc. Autonomic instability is most acute in the first
few days to weeks of the injury.

110. A child presents wit bilious vomiting on day 2 of life and the mother gives
history of polyhydramnios. The radiological imaging is shown. What’s your
probable diagnosis?

a. Duodenal atresia b. Hypertrophic pyloric stenosis


c. Ileal atresia d. Congenital megacolon

Ans. a. Duodenal atresia

Rationale:
• Duodenal atresia is a condition in which the first part of the small bowel (the
duodenum) has not developed properly. It is not open and cannot allow the passage of
stomach contents.
• The hallmark of duodenal obstruction is bilious vomiting without abdominal
distention, which is usually noted on the 1st day of life.
• A history of polyhydramnios is present in half of the pregnancies and is caused by a
failure of absorption of amniotic fluid in the distal intestine.
• The diagnosis is suggested by the presence of a “double bubble sign” on plain
abdominal radiographs. The appearance is caused by a distended and gas-filled
stomach and proximal duodenum.

111. When prioritizing care, which of the following clients should the nurse Olivia
assess first?
a. A 33-year-old client with a recent diagnosis of Guillain-Barre syndrome
b. A 50-year-old client with diverticulitis
c. A 17-year-old client 24-hours post appendectomy
d. A 50-year-old client 3 days post myocardial infarction

Ans. a. A 33-year-old client with a recent diagnosis of Guillain-Barre syndrome


National NORCET Test 6 63

Rationale:
• Guillain-Barre syndrome is characterized by ascending paralysis and potential
respiratory failure. The order of client assessment should follow client priorities, with
disorder of airways, breathing, and then circulation.
• Diverticula are small, bulging pouches that can form in the lining of the digestive
system. When one or more of the pouches become inflamed, and in some cases infected,
that condition is known as diverticulitis. There’s no evidence to suggest perforation for
the client with diverticulitis as a priority of care.
• The client who is post appendectomy has no signs of hemorrhage or unstable vital signs.
Possible complications of appendectomy are bleeding, wound infection, peritonitis,
blocked bowels, and injury to nearby organs.
• There’s no information to suggest the post myocardial infarction client has an
arrhythmia or other complication. About 90% of patients who have an acute MI
develop some form of cardiac arrhythmia during or immediately after the event.

112. A nurse assigned to the emergency department evaluates a patient who


underwent fiberoptic colonoscopy 18 hours previously. The patient reports
increasing abdominal pain, fever, and chills. Which of the following conditions
poses the most immediate concern?
a. Colon cancer
b. Bowel perforation
c. Diverticulitis
d. Viral gastroenteritis

Ans. b. Bowel perforation

Rationale:
• Bowel perforation is the most serious complication of fiberoptic colonoscopy.
• Important signs include progressive abdominal pain, fever, chills, and tachycardia,
which indicate advancing peritonitis. Bowel perforation results from insult or injury to
the mucosa of the bowel wall resulting from a violation of the closed system.
• Colon cancer does not cause these symptoms. Tumor location on clinical presentation
can be separated on left-sided with more changes in bowel habits and hematochezia,
and right-sided with obscured anemia impacting on late stage at diagnosis. The provider
should perform a thorough physical examination for signs of ascites, hepatomegaly,
and lymphadenopathy.
• Diverticulitis may cause pain, fever, and chills, but is far less serious than perforation
and peritonitis.
• Several different viruses including rotavirus, norovirus, adenovirus, and astroviruses
account for most cases of acute viral gastroenteritis. Most are transmitted via the fecal-
oral route, including contaminated food and water. Transmission has also been shown
to occur via fomites, vomitus, and possibly airborne methods.
64 National NORCET Test 6

113. A female client asks for information about osteoarthritis. Which of the
following statements about osteoarthritis is correct?
a. Osteoarthritis is a rare form of arthritis
b. Osteoarthritis affects people over 60
c. Osteoarthritis is rarely debilitating
d. Osteoarthritis is the most common form of arthritis

Ans. d. Osteoarthritis is the most common form of arthritis

Rationale:
• Osteoarthritis is the most common form of arthritis and can be extremely debilitating.
• It can afflict people of any age, although most are elderly.
• Women are more likely to develop Osteoarthritis than men, especially after the age of
50.

114. During a home visit, a client with AIDS tells the nurse that he has been exposed
to measles. Which action by the nurse is most appropriate?
a. Administer an antiviral
b. Administer an antibiotic
c. Tell the client that he should remain in isolation for 2 weeks
d. Contact the physician for an order for immune globulin

Ans. d. Contact the physician for an order for immune globulin

Rationale:
• Measles is caused by morbillivirus, which is mostly seen in the winter and spring. It’s
spread from one child to another through direct contact with discharge from the nose
and throat.
• The client who is immunosuppressed and is exposed to measles should be treated with
medications to boost his immunity to the virus. If the patient knows that he has been
exposed to measles and his CD4 count is less than 200, he should talk to his doctor
about whether post-exposure prophylaxis (PEP) with immunoglobulin may be an
option. PEP may provide some protection or lessen the severity of infection if it occurs.
If the CD4 count is 200 or greater, PEP can also include getting the MMR vaccine.
Ideally, PEP should be administered within 72 hours of exposure to measles.
• Option A: Antivirals would not be as effective as immunoglobulins for the client with
AIDS. Early identification and antiretroviral treatment of HIV-infected infants and
children are critical to maximizing measles vaccine immunogenicity and providing
protection against other HIV-related complications.
• Option B: Antibiotics may not be an effective treatment. One important characteristic
of measles infection is that it produces more serious illness and increased mortality
among immunocompromised individuals, primarily those with defects in T-cell
immunity. Because >90% of the human immunodeficiency virus (HIV)–infected
children live in regions where measles is still endemic, achieving high rates of measles
vaccine coverage is especially important among these populations to suppress excess
measles-associated morbidity and mortality.
National NORCET Test 6 65

• Option C: The patient should remain in isolation, but the administration of


immunoglobulin is a priority. The impact of HIV-related immunocompromise and
subsequent effects of antiretroviral therapy (ART) on immune reconstitution and,
ultimately, on vaccine immunogenicity is unclear.

115. A patient has been in the coronary care unit for the past 24 hours with an acute
anterior myocardial infarction. He develops the abnormal rhythm shown below,
although blood pressure remains stable at 110/68. Which of the following is the
best next step in therapy?

a. Perform cardioversion b. Give propranolol


c. Arrange for pacemaker placement d. Give digoxin

Ans. c. Arrange for pacemaker placement

Rationale:
• The ECG shows complete heart block. Although at first glance the P waves and QRS
complexes may appear related, on closer inspection they are completely independent
of each other (i.e. dissociated). Complete heart block in the setting of acute myocardial
infarction requires temporary (and often permanent) transvenous pacemaker
placement.
• You would certainly want to avoid digoxin, beta-blockers, or any other medication that
promotes bradycardia.
• There is no indication on this strip for cardioversion such as for atrial fibrillation/
flutter or ventricular tachycardia/fibrillation.

116. Which of the following conditions most commonly results in Coronary Artery
Disease (CAD)?
a. MI b. Atherosclerosis
c. Renal failure d. DM

Ans. b. Atherosclerosis

Rationale:
• Coronary artery disease is the narrowing or blockage of the coronary arteries. This
condition is usually caused by atherosclerosis. Atherosclerosis is the build-up of
cholesterol and fatty deposits (called plaques) inside the arteries.
• Myocardial infarction is commonly a result of CAD. Myocardial infarction occurs
when a coronary artery is so severely blocked that there is a significant reduction or
break in the blood supply, causing damage or death to a portion of the myocardium
(heart muscle).
66 National NORCET Test 6
• Renal failure doesn’t cause CAD, but the two conditions are related. Chronic kidney
disease (CKD) accelerates the course of coronary artery disease, independent of
conventional cardiac risk factors. In addition, CKD has been shown to confer inferior
clinical outcomes following successful coronary revascularisation, which may be offset
by arterial grafting.
• DM is a risk factor for CAD but isn’t the most common cause. Near-normal glycemic
control for a median of 3.5 to 5 years does not reduce cardiovascular events. Thus, the
general goal of HbA1c <7% appears reasonable for the majority of patients. Iatrogenic
hypoglycemia is the limiting factor in the glycemic management of diabetes and is an
independent cause of excess morbidity and mortality.

117. A nurse is caring for a patient with peripheral vascular disease (PVD). The
patient complains of burning and tingling of the hands and feet and cannot
tolerate touch of any kind. Which of the following is the most likely explanation
for these symptoms?
a. Inadequate tissue perfusion leading to nerve damage
b. Fluid overloads leading to compression of nerve tissue
c. Sensation distortion due to psychiatric disturbance
d. Inflammation of the skin on the hands and feet

Ans. a. Inadequate tissue perfusion leading to nerve damage

Rationale:
• Patients with peripheral vascular disease often sustain nerve damage as a result of
inadequate tissue perfusion. Intermittent claudication results when blood flow distal
to the occlusion is sufficiently compromised, resulting in fixed oxygen delivery that
is unable to match oxygen demand. The most severe form of PAD is critical limb
ischemia, which is defined as limb pain at rest or impending limb loss.
• Fluid overload is not characteristic of PVD. Atherosclerotic plaque builds up slowly
over decades within the wall of the vessel. Plaque accumulation results in vascular
stenosis and frequent vascular dilation to maximize end-organ perfusion.
• There is nothing to indicate psychiatric disturbance in the patient. The overall prognosis
of patients with peripheral vascular disease must take into account patient risk factors,
cardiovascular health, and disease severity.
• Skin changes in PVD are secondary to decreased tissue perfusion rather than primary
inflammation. Examination of the limbs should involve assessment for pulselessness,
pallor, muscular atrophy, cool or cyanotic skin, or pain with palpation.

118. Nurse Reena is assessing a client with Cushing’s syndrome. Which observation
should the nurse report to the physician immediately?
a. An irregular apical pulse
b. Pitting edema of the legs
c. Frequent urination
d. Dry mucous membranes

Ans. a. An irregular apical pulse


National NORCET Test 6 67

Rationale:

• Apical pulse is auscultated with a stethoscope over the chest where the heart’s mitral
valve is best heard. In infants and young children, the apical pulse is located at the
fourth intercostal space at the left midclavicular line.
• Because Cushing’s syndrome causes aldosterone overproduction, which increases
urinary potassium loss, the disorder may lead to hypokalemia. Therefore, the nurse
should immediately report signs and symptoms of hypokalemia, such as an irregular
apical pulse, to the physician.
• Edema is an expected finding because aldosterone overproduction causes sodium and
fluid retention.
• Frequent urination signals dehydration, which isn’t associated with Cushing’s
syndrome.
• Dry mucous membranes is not a symptom of Cushing’s syndrome. Thinning of the
skin and mucous membranes occur because cortisol causes the breakdown of some
dermal proteins along with the weakening of small blood vessels.

119. A 62-year-old male client was in a motor vehicle accident as an unrestrained


driver. He’s now in the emergency department complaining of difficulty of
breathing and chest pain. On auscultation of his lung field, no breath sounds
are present in the upper lobe. This client may have which of the following
conditions?
a. Bronchitis b. Pneumothorax
c. Pneumonia d. Tuberculosis (TB)

Ans. b. Pneumothorax

Rationale:
• Pneumothorax is defined as the presence of air or gas in the pleural cavity, which can
impair oxygenation and/or ventilation.
• Rhonchi is heard with bronchitis.
• From the trauma the client experienced, it’s unlikely he has pneumonia. Pneumonia may
produce crackling, bubbling, and rumbling sounds when inhaling upon auscultation.
• Bronchial breath sounds with TB would be heard.
68 National NORCET Test 6

120. Identify the structure shown in the image below:-

a. Artery
b. Venules
c. Capillary
d. Arterioles

Ans. c. Capillary

Rationale:
• The given structure is the capillary.
• The wall of the capillary is formed mainly by endothelial cells that are lined on
the outside by basal lamina made of glycoproteins. There is no adventitia which
differentiates it from a venule.
• The capillary can be classified into two types:-
Ĕ Continuous capillaries: The edges of endothelial cells fuse completely with those
of adjoining cells to form a continuous wall. The exchange of material between the
blood and endothelial cells or through the intercellular material adjoining the cells.
Ĕ Fenestrated capillaries: The walls of these capillaries have apertures in their
endothelial lining which are closed by a thin diaphragm. The fenestration may
also be present as pores, which passes through the entire thickness of the cell. The
exchange of material occurs by diffusion through these fenestrae. They are mainly
seen in renal glomeruli, intestinal villi, endocrine glands, and pancreas.

121. A leukemia patient has a relative who wants to donate blood for transfusion.
Which of the following donor medical conditions would make him an
inappropriate donor?
a. Cholecystitis requiring cholecystectomy one year previously
b. Asymptomatic diverticulosis
c. A history of hepatitis C five years previously
d. Crohn’s disease in remission

Ans. c. A history of hepatitis C five years previously


National NORCET Test 6 69

Rationale:
• Hepatitis C is a viral infection transmitted through bodily fluids, such as blood,
causing inflammation of the liver. Patients with hepatitis C may not donate blood
for transfusion due to the high risk of infection in the recipient. Transmission can
be parenteral, perinatal, and sexual, with the most common mode being the sharing
of contaminated needles among IV drug users. Also, other high-risk groups include
people who require frequent blood transfusions and organ transplantation of organs
from infected donors.
• Cholecystitis is inflammation of the gallbladder that occurs most commonly because
of an obstruction of the cystic duct by gallstones arising from the gallbladder
(cholelithiasis).
• Diverticular disease (diverticulosis, diverticulitis) is a general term that refers to the
presence of diverticula, small pouches in the large intestinal (colonic) wall.
• Crohn’s disease is an idiopathic, chronic inflammatory process that can affect any part
of the gastrointestinal tract from the mouth to the anus. Crohn’s disease is believed
to be the result of an imbalance between pro-inflammatory and anti-inflammatory
mediators.

122. What is the most common cause of an abdominal aortic aneurysm (AAA)?
a. Atherosclerosis
b. Diabetes mellitus
c. Hypertension
d. Syphilis

Ans. a. Atherosclerosis

Rationale:
• An abdominal aortic aneurysm is an enlarged area in the lower part of the major vessel
that supplies blood to the body (aorta). The aorta runs from your heart through the
center of your chest and abdomen.
• Smoking is the most common cause of an abdominal aortic aneurysm as well as many
other health problems.
• Atherosclerosis accounts for 75% of all abdominal aortic aneurysms. Plaques build up
on the wall of the vessel and weaken it, causing an aneurysm.
• Diabetes mellitus does not directly cause an abdominal aortic aneurysm.
• Hypertension accounts for 1-15% of cases of an abdominal aneurysm.
• Syphilis is not related to AAA.

123. Nurse Priyanka is teaching a client recovering from Addisonian crisis about
the need to take fludrocortisone acetate and hydrocortisone at home. Which
statement by the client indicates an understanding of the instructions?
a. “I’ll take my hydrocortisone in the late afternoon, before dinner.”
b. “I’ll take all of my hydrocortisone in the morning, right after I wake up.”
c. “I’ll take two-thirds of the dose when I wake up and one-third in the late afternoon.”
d. “I’ll take the entire dose at bedtime.”

Ans. c. “I’ll take two-thirds of the dose when I wake up and one-third in the late afternoon.”
70 National NORCET Test 6

Rationale:
• Addisonian crisis, also known as adrenal crisis or acute adrenal insufficiency is
an endocrinologic emergency with a high mortality rate secondary to physiologic
derangements from an acute deficiency of the adrenal hormone cortisol, requiring
immediate recognition and treatment to avoid death.
• An Addisonian crisis is a life-threatening situation that results in low blood pressure,
low blood levels of sugar and high blood levels of potassium
• Hydrocortisone, a glucocorticoid, should be administered according to a schedule
that closely reflects the body’s own secretion of this hormone; therefore, two-thirds of
the dose of hydrocortisone should be taken in the morning and one-third in the late
afternoon. This dosage schedule reduces adverse effects.
• Option A: Taking the medicine at this schedule may cause adverse effects.
• Option B: The dosage should be according to the imitation of when the body secretes
glucocorticoids.
• Option D: Taking an entire dose might cause severe adverse effects.

124. For a client with Graves’ disease, which nursing intervention promotes comfort?
a. Limiting intake of high-carbohydrate foods
b. Restricting intake of oral fluids
c. Maintaining room temperature in the low-normal range
d. Placing extra blankets on the client’s bed

Ans. c. Maintaining room temperature in the low-normal range

Rationale:
• Graves’ disease is an immune system disorder that results in the overproduction of
thyroid hormones (hyperthyroidism).
• Although a number of disorders may result in hyperthyroidism, Graves’ disease is a
common cause. Thyroid hormones affect many body systems, so signs and symptoms
of Graves’ disease can be wide ranging.
• Graves’ disease causes signs and symptoms of hypermetabolism, such as heat
intolerance, diaphoresis, excessive thirst and appetite, and weight loss. To reduce heat
intolerance and diaphoresis, the nurse should keep the client’s room temperature in the
low-normal range.
• Option A: To provide needed energy and calories, the nurse should encourage the
client to eat high-carbohydrate foods.
• Option B: To replace fluids lost via diaphoresis, the nurse should encourage, not
restrict, intake of oral fluids.
• Option D: Placing extra blankets on the bed of a client with heat intolerance would
cause discomfort.
National NORCET Test 6 71

125. Which of the following Anti-arrhythmic drug causes the given below skin
flushing?

a. Digitalis b. Amiodarone
c. Lidocaine d. Verapamil

Ans. b. Amiodarone

Rationale:
• Amiodarone is an antiarrhythmic medication used to treat and prevent a number
of types of irregular heartbeats. This includes ventricular tachycardia, ventricular
fibrillation, and wide complex tachycardia, as well as atrial fibrillation and paroxysmal
supraventricular tachycardia.
• Amiodarone is known to cause cutaneous and systemic side effects.
• The most common cutaneous side effect is photosensitivity. Blue–gray discoloration
occurs on body areas when exposed to sunlight.
• The disappearance of amiodarone-related skin discoloration may occur within months
or years.

126. A patient admitted to the hospital with myocardial infarction develops severe
pulmonary edema. Which of the following symptoms should the nurse expect
the patient to exhibit?
a. Bradycardia b. Slow, deep respirations
c. Air hunger d. Stridor

Ans. c. Air hunger

Rationale:
• Patients with pulmonary edema experience air hunger, anxiety, and agitation.
• Patients usually present with shortness of breath, which may be acute in onset (from
minutes to hours) or gradual in onset occurring over hours to days, depending upon
the etiology of pulmonary edema.
• Tachypnea is usually present, with the patient gasping for breath. Dyspnea and
tachypnea are usually present, and may be associated with the use of accessory
muscles of respiration. After initial airway clearance, oxygenation assessment, and
maintenance, management mainly depends upon presentation and should be tailored
from patient to patient.
• Respiration is fast and shallow and heart rate increases.
72 National NORCET Test 6
• Tachycardia and hypotension may be present along with jugular venous distention.
Auscultation of the heart helps to differentiate between the various causes of valvular
lesions causing pulmonary edema.
• Stridor is noisy breathing caused by laryngeal swelling or spasm and is not associated with
pulmonary edema. Fine crackles are usually heard at the bases of lungs bilaterally, and
progress apically as the edema worsens. Ronchi and wheeze may also be presenting signs.

127. Nurse Reena is assigned to care for a client who has returned to the nursing
unit after left nephrectomy. Nurse Reena’s highest priority would be…
a. Temperature b. Hourly urine output
c. Able to sips clear liquid d. Able to turn side to side

Ans. b. Hourly urine output

Rationale:
• After nephrectomy, it is necessary to measure urine output hourly. This is done to
assess the effectiveness of the remaining kidney, also to detect renal failure early.
• Monitoring temperature regularly is recommended, but not the highest priority.
• The client would be allowed to take sips of clear liquid as recommended by the
physician. The doctor will give the client more detailed instructions about post-
operative activities, restrictions and diet.
• The client would be able to turn to his sides with some assistance after surgery. The
client will probably remain in the hospital for 1 to 7 days, depending on the method
of surgery used. He will be encouraged to return to light activities as soon as he feels
up to it. Strenuous activity and heavy lifting should be avoided for 6 weeks following
the procedure.

128. Which of the following recurring conditions most commonly occurs in clients
with cardiomyopathy?
a. Heart failure b. DM
c. MI d. Pericardial effusion

Ans. a. Heart failure

Rationale:
• Cardiomyopathy is a disease of the heart muscle that makes it harder for your heart
to pump blood to the rest of your body. Cardiomyopathy can lead to heart failure.
The main types of cardiomyopathy include dilated, hypertrophic and restrictive
cardiomyopathy.
• Because the structure and function of the heart muscle is affected, heart failure most
commonly occurs in clients with cardiomyopathy. Heart failure can occur when the
heart muscle is weak (systolic failure) or when it is stiff and unable to relax normally
(diastolic failure). Cardiomyopathy, which means “disease of the heart muscle,” is one
of many causes of heart failure.
• Type 1 and type 2 diabetic patients are at increased risk of cardiomyopathy and
heart failure is a major cause of death for these patients. Cardiomyopathy in diabetes
is associated with a cluster of features including decreased diastolic compliance,
interstitial fibrosis, and myocyte hypertrophy.
National NORCET Test 6 73

• Myocardial infarction results from prolonged myocardial ischemia due to reduced


blood flow through one of the coronary arteries.
• Pericardial effusion is most predominant in clients with pericarditis. Pericardial
effusion puts pressure on the heart, affecting the heart’s function. If untreated, it can
lead to heart failure or death.

129. The physician orders laboratory tests to confirm hyperthyroidism in a female


client with classic signs and symptoms of this disorder. Which test result would
confirm the diagnosis?
a. An increase in the TSH level after 30 minutes during the TSH stimulation test
b. No increase in the thyroid-stimulating hormone (TSH) level after 30 minutes during
the TSH stimulation test
c. Below-normal levels of serum triiodothyronine (T3) and serum thyroxine (T4) as
detected by radioimmunoassay
d. A decreased TSH level

Ans. b. No increase in the thyroid-stimulating hormone TSH level after 30 minutes during the
TSH stimulation test

Rationale:
• Hyperthyroidism (overactive thyroid) occurs when your thyroid gland produces
too much of the hormone thyroxine. Hyperthyroidism can accelerate your body’s
metabolism, causing unintentional weight loss and a rapid or irregular heartbeat.
• In the TSH test, failure of the TSH level to rise after 30 minutes confirms
hyperthyroidism.
• Option A: If the TSH level rises after 30 minutes, then the client has no hyperthyroidism.
• Option C: Below-normal levels of T3 and T4, as detected by radioimmunoassay, signal
hypothyroidism. A below-normal T4 level also occurs in malnutrition and liver disease
and may result from the administration of phenytoin and certain other drugs.
• Option D: A decreased TSH level indicates a pituitary deficiency of this hormone.

130. During which interval on the electrocardiogram (ECG) below does the aortic
valve close?

w
a. A b. B
c. C d. D

Ans. d. D
74 National NORCET Test 6

Rationale:
• The aortic valve closes when the pressure within the ventricle falls below the pressure
within the aorta. This occurs when the ventricular muscle begins to relax. Relaxation
begins at the end of the ventricular action potential, which corresponds to the end of
the T wave (segment D) on the ECG recording.

131. Mrs. Arora is diagnosed with osteoporosis. Which electrolytes are involved in
the development of this disorder?
a. Phosphorus and potassium b. Calcium and sodium
c. Potassium and sodium d. Calcium and phosphorous

Ans. d. Calcium and phosphorous

Rationale:
• Osteoporosis: A condition in which bones become weak and brittle.
• Osteoporosis related fractures most commonly occur in the hip, wrist or spine. Bone is
living tissue that is constantly being broken down and replaced.
• The main cause of osteoporosis is bone loss due to a drop in your body’s estrogen levels.
Estrogen is a hormone that helps build and maintain your bones. The most common
cause of estrogen loss in women is menopause.
• In osteoporosis, bones lose calcium and phosphate salts, becoming porous, brittle, and
abnormally vulnerable to fracture.
• Potassium is not involved in osteoporosis. Potassium salts aid in bone health. A study,
published in the journal Osteoporosis International, also revealed that high intake of
potassium salts significantly reduces the excretion of calcium and acid in urine.
• Sodium increases calcium excretion and higher calcium excretion is associated with
lower bone mineral density, a predictor of osteoporotic fractures.

132. Mr. Dubey is diagnosed with hypermagnesemia. Symptoms of hiscondition


may include:
a. Hypertension b. Tachycardia
c. Hyperactive deep-tendon reflex d. Cardiac arrhythmias

Ans. d. Cardiac arrhythmias

Rationale:
• Cardiac arrhythmias are associated with hypermagnesemia.
• Hypertension, tachycardia, and hyperactive reflexes are signs of hypomagnesemia.

133. A nurse is evaluating a postoperative patient and notes a moderate amount of


serous drainage on the dressing 24 hours after surgery. Which of the following
is the appropriate nursing action?
a. Remove the dressing and leave the wound site open to air
b. Notify the surgeon about evidence of infection immediately
c. Change the dressing and document the clean appearance of the wound site
d. Leave the dressing intact to avoid disturbing the wound site

Ans. c. Change the dressing and document the clean appearance of the wound site
National NORCET Test 6 75

Rationale:
• A moderate amount of serous drainage from a recent surgical site is a sign of normal
healing. Serous drainage is clear, thin, and watery. The production of serous drainage
is a typical response from the body during the normal inflammatory healing stage.
• The surgical site is typically covered by gauze dressings for a minimum of 48-72 hours
to ensure that initial healing has begun. Changing the dressing less, allows the wound
bed to be left undisturbed, which allows for the migration of new cells. When wound
beds are left undisturbed in an optimal moist environment, they are able to heal at a
faster rate.
• Purulent drainage would indicate the presence of infection. Purulent drainage is milky,
typically thicker in consistency, and can be gray, green, or yellow in appearance. If the
fluid becomes very thick, this can be a sign of infection. Yet, if there is a large amount
of serous drainage, it can be the result of a high bioburden count.
• A soiled dressing should be changed to avoid bacterial growth and to examine the
appearance of the wound. Overall, it should be noted that the dressing selection should
be based on the individual patient and wound characteristics. If the wound is not in the
normal inflammatory phase of healing, the clinician must investigate what is the root
cause and how to manage the drainage.

134. A female client arrives at the emergency department with chest and stomach
pain and a report of black tarry stool for several months. Which of the following
orders should the nurse Priyanka anticipate?
a. Prothrombin time, partial thromboplastin time, fibrinogen and fibrin split product
values
b. Cardiac monitor, oxygen, Creatine kinase and lactate dehydrogenase levels
c. Electroencephalogram, alkaline phosphatase, and aspartate aminotransferase levels,
basic serum metabolic panel
d. Electrocardiogram, complete blood count, testing for occult blood, comprehensive
serum metabolic panel

Ans. d. Electrocardiogram, complete blood count, testing for occult blood, comprehensive
serum metabolic panel

Rationale:
• An electrocardiogram evaluates the complaints of chest pain, laboratory tests determine
anemia, and the stool test for occult blood determines blood in the stool.
• Prothrombin time, partial thromboplastin time, fibrinogen and fibrin split products
are measured to verify bleeding dyscrasias.
• Cardiac monitoring, oxygen, and creatine kinase, and lactate dehydrogenase levels
are appropriate for a cardiac primary problem. A basic metabolic panel and alkaline
phosphatase and aspartate aminotransferase levels assess liver function.
• An electroencephalogram evaluates brain electrical activity.
76 National NORCET Test 6

135. The term “pink puffer” depicted in the image refers to the client with which of
the following conditions?

a. Asthma
b. Adult respiratory distress syndrome (ARDS)
c. Chronic obstructive bronchitis
d. Emphysema

Ans. d. Emphysema

Rationale:
• Emphysema is a lung condition that causes shortness of breath. In people with
emphysema, the air sacs in the lungs (alveoli) are damaged. Over time, the inner walls
of the air sacs weaken and rupture — creating larger air spaces instead of many small
ones.
• Because of the large amount of energy it takes to breathe, clients with emphysema are
usually cachectic. They’re pink and usually breathe through pursed lips, hence the term
“puffer.”
• Clients with asthma don’t have any particular characteristics.
• Clients with ARDS are usually acutely short of breath.
• Clients with chronic obstructive bronchitis are bloated and cyanotic in appearance.

MENTAL HEALTH NURSING


136. Which of the following is NOT true about sleep?
a. It consists of REM and non-REM sleep.
b. Sleep cycles are controlled by the suprachiasmatic nucleus.
c. Most growth hormones are produced during stage 3 and 4 of sleep.
d. Dreams occur during non-REM sleep.

Ans. d. Dreams occur during non-REM sleep


National NORCET Test 6 77

Rationale:
• REM is when dreaming, nightmares, and penile/clitoral tumescence occur; may serve
memory processing function.
NREM and REM Sleep
Physiological Process NERM (Slow wave Sleep) REM (Paradoxical Sleep)
Time spent 75-80% of total sleep duration 20-25 % of total sleep duration
Brain activity Decreases from wakefulness Increases in motor and sensory
areas, while other areas arc similar
to NERM
Heart rate, BP, Decreases from wakefulness Increases and varies compared to
Blood flow to Brain, NERM
Respiration
Sympathetic nerve Decreases from wakefulness Increases significantly from
activity wakefulness
Muscle tone Similar to wakefulness Absent in all muscle except facial and
extra ocular muscles.
Body temperature Is regulated at lower set point Is not regulated; no shivering or
than wakefulness; shivering sweating; temperature drifts toward
initiated at lower temperature that of the local environment.
than during wakefulness
Sexual arousal Occurs infrequently Greater than NERM
EEG Depends on the stages of sleep • Rapid, low—voltage beta like EEG
activity, which resembles that
seen in the awake (Paradoxical
sleep).
• P-G-O (Ponto-Geniculo-Occipital)
spikes (Cholinergic) are seen.
Dreams Non-recallable dream Recallable dream
(So called Dream)

Stages of NREM Sleep


Recent Old Stage % of total EEG changes Behavioral observation
Stages sleep duration
N1 Stage-1 2-5 % Low amplitude, high • Easily aroused by
(Light sleep) frequency EEG activity moderate stimuli
• Continuous lack of
awareness
N2 Stage-2 45-55% Appearance of sleep • Further lack of sensitivity
(True sleep) Spindles (Alpha to activation and arousal
like 10-14/sec,
50µV amplitude), K
complexes

Contd…
78 National NORCET Test 6

Recent Old Stage % of total EEG changes Behavioral observation


Stages sleep duration
N3 Stage-3 10-15% Sleep spindles • Sleep deepens
superimposed on a
background of δ
waves
Stage-4 Slow high voltage δ • Activation & arousal
(Deep sleep) waves occurs with vigorous
stimulation
• When awakened
person does not report
dreaming

Sleep Disorder
NERM Sleep Disorder
Sleep walking Stage-3 & 4 (Stag N3) can occur during REM sleep also.
(Somnambulism)
Somniloquy Stage – 1 & 2 mainly but can possible in all.
(Sleep talking)
Bruxism Stage 1 and 2 mainly (Stage 2 > Stage 1). May be in ECM and stage 3 and
Stage 4.
Nocturnal enuresis Al stages of NERM and REM except Stage-1 (Maximum during Stage-2)
Night terrors Transition from stage 3 to Stage-4 (Stage N3)
REM Sleep Disorder
Narcolepsy REM Sleep
Night mare REM Sleep

137. The normal value of serum lithium test is _________


a. 0.6 –1.2 mmol/L
b. 0.10 – 3.2 mmol/L
c. 1–1.8 mmol/L
d. 1.2–2.2 mmol/L

Ans. a. 0.6 –1.2 mmol/L

Rationale:
• Lithium is used for treatment of acute episodes (both mania and depression) as well as
prophylaxis in bipolar disorder.
• Lithium has a narrow therapeutic index and therapeutic drug monitoring is required.
The effective serum concentration for treatment of acute mania is 1.0-1.5 mEq/dL. The
serum concentration required for maintenance treatment is 0.6-1.2 mEq/dL.
National NORCET Test 6 79

Plasma concentration of Relevance


Lithium (mEq/L)
1-1.5 Acute attack of mania
0.6-1 Prophylaxis of mania, BPD
>2 Indicator of toxicity
>3 Permanent neurological damage.
Absolute indication for dialysis
>4 Death

138. A client sleeps between conversations. It is described as:


a. Sleep apnea
b. Narcolepsy
c. Somnambulism
d. Insomnia

Ans. b. Narcolepsy

Rationale:
• Narcolepsy: Primary characteristic is excessive daytime sleepiness. Sudden sleep
attacks (repeated) during the day for at least 3 months.
• Sleep walking: Somnambulism
• Somniloquy: Sleep talking
• Bruxism: Tooth grinding
• Insomnia: Problems falling asleep and/or staying asleep.

139. CAGE questionnaire is used for which of the following behavioral disorders?
a. Alcohol screening
b. Heroine screening
c. Opioid screening
d. Naloxone screening

Ans. a. Alcohol screening

Rationale:
• The CAGE questionnaire is a series of four questions that doctors can use to check for
signs of possible alcohol dependency.
• The questions are designed to be less obtrusive than directly asking someone if they
have a problem with alcohol.
• CAGE Questions for Alcohol Use:
Ĕ Have you ever felt you needed to Cut down on your drinking?
Ĕ Have people Annoyed you by criticizing your drinking?
Ĕ Have you ever felt Guilty about drinking?
Ĕ Have you ever felt you needed a drink first thing in the morning (Eye-opener) to
steady your nerves or to get rid of a hangover?
80 National NORCET Test 6

140. Among the following, which is not a sign of depression?


a. Anhedonia b. Irritability
c. Restless d. Hopeful

Ans. d. Hopeful

Rationale:
Signs of depression:
• Sleep
• Loss of Interest
• Guilt
• Low Energy
• Poor Concentration
• Loss of Appetite
• Psychomotor Ds
• Suicidal ideation
Ĕ Hopelessness
Ĕ Helplessness
Ĕ Worthlessness

141. In schizophrenia, gradual subtle behavioral changes occur in:


a. The premorbid phase
b. The prodromal phase
c. The onset phases
d. The progressive phase

Ans. b. The prodromal phase

Rationale:
• Symptoms of schizophrenia often present in three phases:
Ĕ Prodromal: Decline in functioning that precedes the first psychotic episode. The
patient may become socially withdrawn and irritable. He or she may have physical
complaints, declining school/work performance, and/or newfound interest in
religion or the occult.
Ĕ Psychotic: Perceptual disturbances, delusions, and disordered thought process/
content.
Ĕ Residual: Occurs following an episode of active psychosis. It is marked by mild
hallucinations or delusions, social withdrawal, and negative symptoms.

142. Parkinson’s disease is characterized by deficiency of_____ neurotransmitter.


a. Dopamine
b. Acetylcholine
c. Serotonin
d. GABA

Ans. a. Dopamine
National NORCET Test 6 81

Rationale:
• Parkinson’s disease: It is caused by a loss of dopaminergic neurons in the substantia
nigra pars compacta, which results in lower levels of dopamine in the CNS.
• Dopamine is usually increased in schizophrenia.
• Neurotransmitter imbalance in depression:
Ĕ Serotonin- Decreased
Ĕ Acetylcholine-increased
Ĕ Histamine- Decreased
Ĕ Norepinephrine- Decreased

143. Most common symptom of alcohol withdrawal is:


a. Body ache b. Tremor
c. Rhinorrhea d. Diarrhea

Ans. b. Tremor

Rationale:
• Signs and symptoms of alcohol withdrawal syndrome include insomnia, anxiety, hand
tremor, irritability, anorexia, nausea, vomiting, autonomic hyperactivity (diaphoresis,
tachycardia, and hypertension), psychomotor agitation, fever, seizures, hallucinations,
and delirium.

144. All of the following are mood stabilizing agents, except:


a. Lithium carbonate b. Fluoxetine
c. Valproic acid d. Carbamazepine

Ans. b. Fluoxetine

Rationale:
• Fluoxetine is antidepressant.
• Selective serotonin reuptake inhibitors are sertraline, paroxetine, fluoxetine,
citalopram, Escitalopram.
• Mood Stabilizers: Lithium, Lamotrigine and Valproic acid.

145. Best test for diagnosis of organic mental disorder is:


a. Bender gestalt test b. Sentence completion test
c. Rorschach test d. Thematic appreciation test

Ans. a. Bender gestalt test

Rationale:
• The Bender Visual-Motor Gestalt Test is a psychological test used by mental health
practitioners that assesses visual-motor functioning, developmental disorders, and
neurological impairments in children aged 3 or moreand adults.
82 National NORCET Test 6

146. What is the purpose of using succinylcholine during the Electroconvulsive


therapy?
a. Reduction in oral secretions b. Muscle relaxant
c. Muscle contraction d. Both (b) and (c)

Ans. b. Muscle relaxant

Rationale:
• During Electro Convulsive Therapy (ECT) patients are often pre-medicated with
atropine, and then given general anesthesia (typically with methohexital) and muscle
relaxants (e.g., succinylcholine).

147. Which of the following are the causes of delirium?


a. Brain tumor b. Head trauma
c. Alcohol consumption d. All of the above

Ans. d. All of the above

Rationale:
Causes of Delirium:
Ĕ Loss of cortical neurons
Ĕ Impaired delivery of substrates/blood to the brain,
Ĕ CNS infection
Ĕ Neurosensory disturbance
Ĕ Metabolic disturbances, drugs (e.g., alcohol, opiates, barbiturates),
Ĕ Renal failure
Ĕ Urinary tract infection
Ĕ Liver disease
Ĕ Hypoxemia
Ĕ Endocrinopathy (e.g., thyroid storm)

148. Which of the following is a symptom of overdose of lithium:


a. Polyuria and excessive drinking of water
b. Weight loss
c. Leukocytopenia
d. Hyporeflexia

Ans. a. Polyuria and excessive drinking water

Rationale:
• Lithium is rapidly absorbed after oral administration with peak serum concentrations
occurring within 1-1.5 hours. Plasma half- life is 1.3 days. Equilibrium is reached
after 5-7 days.
• Early signs of lithium toxicity:
Ĕ Coarse tremor
Ĕ Dysarthria
National NORCET Test 6 83

Ĕ Ataxia
Ĕ GI symptoms
• Later signs
Ĕ Fasciculations, myoclonus, seizures, coma
• Risk factors - renal impairment, low sodium diet, drug interaction and dehydration
• Treatment guidelines
Ĕ Medical emergency
Ĕ Discontinue lithium
Ĕ Vital signs and neurological evaluation
Ĕ Lithium level, Serum electrolytes, renal function test and ECG
Ĕ Gastric lavage with activated charcoal
Ĕ Hemodialysis (if lithium more than 4 mEq/L)

149. Schizotypal disorder is characterized by:


a. Manic attacks b. Severe social anxiety
c. Suicidal tendency d. Hallucinations

Ans. b. Severe social anxiety

Rationale:
• Schizotypal is a Cluster A personality disorder that also falls on the schizophrenia
spectrum. May include brief psychotic episodes (e.g., delusions) that are less frequent
and severe than in schizophrenia.
• Clinical Findings are:-
Ĕ Odd thought patterns, behavior, and beliefs. Magical thinking (not as severe as
schizophrenics), interpersonal awkwardness
Ĕ Paranoid ideation
Ĕ Ideas of reference
Ĕ Inappropriate/constricted affect

Personality Disorders

Cluster A: Odd or Eccentric


Types Definition Epidemiology Associated
Defenses
Paranoid Feelings of persecution; feels • Men > women Projection
that others are conspiring to • Increased incidence
harm them; suspicious in families with
schizophrenia
Schizoid Isolated lifestyle; has no • Men > women
longing for others (“loner”) • Increased incidence ------
in families with
schizophrenia
Schizotypal Eccentric behavior, thought, • Prevalence is 3% ------
and speech • Men > women
84 National NORCET Test 6

Cluster B: Dramatic and Emotional


Types Definition Epidemiology Associated Defenses
Histrionic Excessive emotion and • Women > men • Regression
attention seeking • Underdiagnosed in • Somatization
men • Conversion
• Dissociation
Narcissistic Grandiose; over Common Fixation at sub phase of
concerned with issues separation/individualization
of self-esteem
Borderline Instability of mood, • Women > men • Splitting
self-image, and • ↑ mood disorders in • Projective identification
relationships families • Dissociation
• Passive-aggression
Antisocial Does not recognize the Prevalence: 3% in men; Superego lacunae
rights of others 1% in women

Cluster C: Anxious and Fearful


Types Definition Epidemiology Associated Defenses
Avoidant Shy or timid; • Common Avoidance
fears rejection • Possible deforming illness
Dependent Dependent, • Common ------------
submissive • Women > men
• May end up as abused spouse
Obsessive Perfectionistic • Men > women • Isolation
compulsive and inflexible, • ↑ concordance in identical • Reaction formation
orderly, rigid twins • Undoing
• Intellectualization

150. A person thinks that somebody follows him because he had secret paper in his
briefcase:
a. Delusion of persecution b. Idea of reference
c. Idea of suspect d. Delusion of grandeur

Ans. a. Delusion of persecution

Rationale:
• Persecutory delusions are a form of paranoia. They’re often seen in schizophrenia and
other mental illnesses, like schizoaffective disorder and post-traumatic stress disorder.
• When someone experiences persecutory delusions, they have convinced that someone
is mistreating, conspiring against, or planning to harm you or your loved one.

151. According to psychosocial theory of Erikson, which behavior is seen in toddler


age group?
a. Autonomy vs. shame b. Generativity vs. stagnation
c. Ego integrity vs. despair d. Trust vs. mistrust

Ans. a. Autonomy vs. shame


National NORCET Test 6 85

Rationale:
• The 8 stage of psychological development was given by Erik Erikson.
• He states that through the course of development there are critical time periods, each
of which poses a particular psychological conflict to the individual. The resolution of
this conflict results in either psychosocial growth or regression.
Stage and Age Goal
Stage-1 • Infancy Trust Vs Mistrust
• Birth-18 months
Stage-2 • Toddler Autonomy Vs Shame and Doubt
• 18 months-3 years
Stage-3 • Pre-school Initiative Vs Guilt
• 3-6 years
Stage-4 • School going Industry Vs Inferiority
• 6-12 YEARS
Stage-5 • Adolescence Identity Vs Role confusion
• 12-18 years
Stage-6 • Young Adulthood Intimacy vs isolation
• 18-35 years
Stage-7 • Middle adulthood Generativity vs Stagnation
• 35-65 years
Stage-8 • Late Adulthood Ego integrity Vs Despair
• > 65 years

152. The nurse is caring for a 58 year old man present with dementia. The nurse
knows that which of the following should be the priority in caring for this
client?
a. Institute various fall prevention measures
b. Provide detailed written instructions
c. Provide the client with emotional support
d. Encourage long periods of daytime sleep

Ans. a. Institute various fall prevention measures

Rationale:
• Elderly patients have slower reflexes, decreasing ability to adapt to new environments
and balance issues which leads to falls. Maintaining patients’ safety a priority; institute
fall prevention measure is required to maintain patient’s safety.
• No detailed written instructions; simple written instruction may serve as helpful
reminders to a patient with dementia.
• Emotional support is important for a patient with dementia; however patient’s safety
takes priority over psychological needs.
• Long periods of day time sleep should be discouraged.
86 National NORCET Test 6

153. The nurse provides care to a 60 year old male patient, who is agitated and
pulling things. Previously he has diagnosed with Alzheimer disease. Which of
the following action should be implemented by the nurse when providing care
to the patient?
a. Place the patient in a room with several other patients.
b. Provide the patient with therapeutic sensory devices
c. Leave the patient alone for a set period of time to reduce environmental stimulation
d. Cohort the patient with another patient who is agitated, because when will calm each
other.

Ans. b. Provide the patient with therapeutic sensory devices

Rationale:
• Patients with Alzheimer disease most commonly pick at items, such as buttons of
clothing or medical devices, which poses a danger to them. Providing them with safety
designed sensory devices serves the need of stimulating the senses as well as their urge
to pick.
• Placing the patient in a room with several other patients can worsen the problem due
to increased stimulation.
• Leaving the patient alone could lead to injuries related to the agitation and picking.
• Cohorting the patient with another agitated patient can worsen the problem due to
increased stimulation.

154. Which of the following is not a risk factor for delusional disorder?
a. Recent immigration b. Social isolation
c. Family history d. Young age

Ans. d. Young age

Rationale:
• Young age is NOT a risk factor for delusional disorders.
• Risk factors for delusional disorders:
Ĕ Advanced age
Ĕ Sensory impairment or isolation
Ĕ Family history
Ĕ Social isolation
Ĕ Recent immigration
Ĕ Personality features (e.g. Unusual interpersonal sensitivity)

155. The hierarchy of needs was described by:


a. Skinner
b. Johann Reli
c. Abraham Maslow
d. Karl Kahlbaum

Ans. c. Abraham Maslow


National NORCET Test 6 87

Rationale:
• The hierarchy of needs was described by Abraham Maslow.
• It is a motivational theory in psychology which has five-tier model of human needs,
often depicted as hierarchical levels within a pyramid.
• Maslow stated that people are motivated to achieve certain needs and that some needs
take precedence over others.

FUNDAMENTAL OF NURSING
156. Nurses who care for the terminally ill apply the theories of Kübler-Ross in
planning care. Place the stages of this process in the order identified by Kübler-
Ross.
I. Acceptance II. Bargaining
III. Depression IV. Anger
V. Denial
a. V-IV-II-III-I b. V-III-IV-II-I
c. V-II-IV-I-III d. V-IV-III-II-I

Ans. a. V-IV-II-III-I

Rationale:
According to Kübler-Ross, individuals who experience a terminal illness go through a
grieving process.
88 National NORCET Test 6

DABDA

Elisabeth Kubler-Ross
Elisabeth Kubler-Ross in her book “On Death and Dying” described five stages of grief
which represent the normal range of feelings people experience when facing death or
significant change in their lives.

157. Ms. Reena, working as a home health care nurse and assigned to providing care
to a terminally ill cancer client. While performing the general assessment, the
client said to Ms. Reena “I’m no longer afraid of dying. I think I’ve made my
peace with everyone, and I’m actually ready to move on.” This reflect client’s
progress to which stage of death and dying according to Kubler ross model?
a. Acceptance b. Ange
c. Bargaining d. Denial

Ans. a. Acceptance

Rationale:
National NORCET Test 6 89

Table: Kubler Ross model/stages of grief


Stages Description
Denial • State of refusal to accept facts, information, reality, etc., relating to the
situation concerned.
• It’s a natural and normal defence mechanism which body start in response to
grief.
• Some people can become locked in this stage when dealing with a traumatic
change that can be ignored.
Anger • After the denial stage pass, the client moved to second stage which is Anger.
It can be manifest in different ways. People dealing with emotional upset can
be angry with themselves, and/or with others, especially those close to them.
Bargaining • Bargaining is the stage where client begins to have an irrational hope even
when the facts say otherwise.
• In the bargaining stage the client bargain to their God for giving one more
chance.
Example: They may agree to live a good life, help the needy, never lie again, or
any number of “good” things if their higher power will only cure them of their
illness.
Depression • In this instance, Depression is not a sign of a mental health condition, instead,
it’s a natural and appropriate response of body to grief.
• During the depression stage, one starts facing the present reality and the
inevitability of the loss experienced.
Acceptance • The last stage of grief identified by Kübler-Ross is acceptance.
• In this stage, person’s emotions may begin to stabilize. You re-enter reality.

158. Which is a psychological response to​grief?


a. Anger b. Fatigue
c. Decreased appetite d. Sleep deprivation

Ans. a. Anger

Rationale:
• According to Kubler Ross model of grieving each person has to go through 5 stages of
grief. Denial---Anger---Bargaining---depression---Acceptance.
• Anger is a psychological response to grief.
• Sleep deprivation, decreased appetite, fatigue are physiological responses to grief.

159. The emotional response to a death is called ________.


a. Depression b. Grieving
c. Bereavement d. Mourning

Ans. b. Grieving

Rationale:
• Grieving: Grieving is the normal process of reacting to the loss. It is defined as internal
emotional response to loss. Loss could include anything like death of loved one, loss of
body part (amputation), loss of job etc.
90 National NORCET Test 6
• Depression: It is a part of grieving explained by Kubler-Ross.
• Bereavement: Bereavement is the period after a loss during which grief is experienced
and mourning occurs.
• Example: Husband died due to cancer. His wife (bereaved: the person who is experience
the grief) will experience the grief start from his husband death till he accept the reality
that now his husband is no longer physically available with her.
• Mourning is the process by which people adapt to a loss. Mourning is also influenced
by cultural customs, rituals, and society’s rules for coping with loss. Example: Funeral

160. You are posted in hospice centre and assisting to the nurse in her daily routine
assessment. You observed that the nurse is applying Kubler-Ross model on a
patient with lung cancer. After assessing the patient, the nurse recorded that
the patient is still in denial phase. You got curious and asked to the nurse how
does the denial phase differ from the acceptance stage? Select the best response
made by the nurse?
a. The acceptance stage occurs before the denial stage.
b. During the denial phase patient is stunned, and not ready to face the reality.
c. During the acceptance stage, the person accepted the reality about the loss.
d. Both b & c

Ans. d. Both b & c

Rationale:

161. A nurse is caring for a terminally ill patient admitted with chronic liver
cirrhosis. The patient says, “I can’t sleep, I keep on thinking about my family
what will they do when I die.” What response by the nurse would be most
appropriate in this situation?
a. Stop thinking about them, you should take proper rest.
b. “What seems to be concerning you the most?”
c. “I will talk to your family on your behalf and explain them about your situation.
d. ” I understand your concern, but I can’t do anything in this.

Ans. b. “What seems to be concerning you the most?”

Rationale:
• Here in the given scenario the best appropriate response made by the nurse will be
1. Asking open ended question (e.g; to ask the patient to describe or elaborate the
situation) to identify the need and solution.
2. Use silence (a therapeutic communication technique) which gives an opportunity
to the patient to share his concern point.
• Whereas other options reflect the non- therapeutic communication technique which
can disturb the Nurse -Patient relationship and doesn’t help the nurse to identify the
patient emotional concern.
National NORCET Test 6 91

162. Identify the dosage form shown in the given below?

a. Suppository b. Pessary
c. Douche d. Respule

Ans. a. Suppository

Rationale:
Option a: Rectal suppositories are solid dosage forms of medication that are inserted into
the rectum. They are manufactured in a torpedo shape with a pointed end (apex) and a
blunt end (see picture above). The blunt end is often concave, forming a useful indentation
for the fingertip to push against.
Option b: A pessary is a soft, flexible device that is placed in the vagina to support the
surrounding structure like bladder, vagina, uterus, and/or rectum. Pessaries are made
in many different shapes and sizes. A pessary is a non-surgical way to treat pelvic organ
prolapse and sometimes incontinence.

Figure 1: Pessaries
Option c: The word ‘’douche’’ is come from a French word meaning ‘’wash’’ or ‘’soak.’ A
douche is a medical device which is used to introduce water to the inside of a part of the
body so that it can be washed or soaked.

Figure 2: Douches
92 National NORCET Test 6
Option d: A Respule is a small plastic container that contains a liquid. The liquid is put into
a machine called a nebuliser. This machine turns the medicine into a fine mist which can
breathe in through a face mask or mouthpiece.

Figure 3: Respule

163. A junior nurse is providing a suppository to her patient. Which step performed
by the junior nurse reflect her poor understanding regarding administration
of suppository?
a. Place the patient in sim’s position.
b. Use K-Y jelly to lubricate the suppository.
c. Insert the suppository about 4 inch.
d. Instruct the patient to walk immediately after administration

Ans. d. Instruct the patient to walk immediately after administration

Rationale:

Procedure of Administration of Suppository


• Place the patient in sim’s position.

Figure 1: Sim’s Position


• Wear gloves
• Lubricate the tip of suppository.
• Encourage the patient to relax by breathing through the mouth to relaxes the external
anal sphincter
• Insert the index finger about 1 inch in adult or ½ inch in children
• Administer suppository with the help of index finger beyond the internal sphincter 4
inch (10cm)
National NORCET Test 6 93

• Press the patient buttocks together for few minutes


• Instruct the patient to remain in the same position at least 5 minutes

Figure 2: Insertion of suppository

164. Which of the following statement is incorrect in relation to suppository?


a. Suppository are used to provide local effect only.
b. Suppository are used to provide local as well as systemic effect.
c. Cocoa butter is useful as a suppository base.
d. Place the patient in sim’s position.

Ans. a. Suppository are used to provide local effect only.

Rationale:
• Cocoa butter (theobroma oil) is useful as a suppository base because of its melting
point.
• Suppositories for local effect are useful in the management of chronic constipation, in
bowel preparation prior to bowel investigations and for the treatment of itching and
pain caused by haemorrhoids as well as for systemic effect.
• Suppository should be provided in sim’s position.

165. A clinical instructor is providing microteaching to the staff on patient


confidentiality. Which of the following statements would be included in the
presentation?
a. Verbal consent is sufficient to allow family members to see a patient’s medical records.
b. All hospital staff may have access to a patient’s medical records.
c. Consent to disclosure is implied when a patient is transferred from one health provider
or facility to another.
d. All of the above

Ans. c. Consent to disclosure is implied when a patient is transferred from one health provider
or facility to another.
94 National NORCET Test 6

Rationale:
• Option c: Consent to disclosure is implied in the transfer from one health provider
to another for Continuity of Care under the Health Insurance Portability and
Accountability Act of 1996 (HIPAA) (a federal law that required the creation of
national standards to protect sensitive patient health information from being disclosed
without the patient’s consent or knowledge) as long as the facility members receiving
the Patient Health Information, or PHI, are directly involved in the care of the patient
• Option a: Consent must be in writing from the patient to allow the family members to
see the patient’s medical record or receive information from care providers: Updates on
the patient’s status should only be given with consent from the patient. This includes all
family members, even a spouse.
• Option b: Under HIPAA laws, only staff operating in the direct care of the patient are
allowed access to a patient’s medical records.

166. Which of the following statement a nurse should follow initially for obtaining
consent from the patient for surgery?
a. Describe the risks involved in the surgery.
b. Explain that obtaining the signature is routine for any surgery.
c. Witness the client’s signature.
d. Determine whether the client’s knowledge level is sufficient to give consent.

Ans. d. Determine whether the client’s knowledge level is sufficient to give consent.

Rationale:
• Option d: Informed consent means the client must comprehend the surgery, the
alternatives, and the consequences.
• Option a: Describing the risk associated with any surgical procedure is not under
nursing domain. This is a surgeon responsibility who will be operating the patient.
• Option b: Although this is true, but here in the question it is clearly stated what will be
the INITIALLY step in obtaining consent. Option b will not be considered as a correct
answer because nurse first need to assess the patient ability to sign the consent form.
• Option c: Although this is true, but the nurse should first assess the client ability to
give consent.

167. The family of an older adult who is aphasic reports to the team leader that the
assigned nurse failed to obtain a signed consent before inserting an indwelling
catheter. What will be the best statement given by the team leader in this
scenario?
a. Procedures for a client’s benefit do not require a signed consent.
b. Clients who are aphasic are incapable of signing an informed consent.
c. A separate signed informed consent for routine treatments is unnecessary.
d. A specific intervention without a client’s signed consent is an invasion of rights.

Ans. c. A separate signed informed consent for routine treatments is unnecessary.


National NORCET Test 6 95

Rationale:
• Option c: This is considered as a routine procedure to meet basic physiologic needs as
per the Maslow’s hierarchy and is covered under the consent form which was already
signed at the time of admission.
• Other options are acting as a distractors here in the given scenario.

168. A client is voluntarily admitted to a psychiatric unit. Later, the client develops
severe pain in the right lower quadrant and is diagnosed as having acute
appendicitis. How should the nurse prepare the client for the appendectomy?
a. Have two nurses witness the client signing the operative consent form.
b. Ensure that the surgeon and the psychiatrist sign for the surgery because it is an
emergency procedure.
c. Ask the client to sign the operative consent form after the client has been informed of
the procedure and required care.
d. Inform the client’s relative that it will be necessary for one of them to sign the consent
form because the client is on a psychiatric unit.

Ans. c. Ask the client to sign the operative consent form after the client has been informed of the
procedure and required care.

Rationale:
• Option c: Because the client is not certified as incompetent, the right of informed
consent is retained.
• Option a: The client can sign the consent, but the client’s signature requires only one
witness.
• Option b & d: Because there is no evidence of incompetence, the client should sign
the consent.

169. What should the nurse consider when obtaining an informed consent from a
17-year-old married girl?
a. If the client is allowed to give consent
b. The client cannot make informed decisions about health care.
c. If the client is permitted to give voluntary consent when parents are not available
d. The client probably will be unable to choose between alternatives when asked to
consent.

Ans. a. If the client is allowed to give consent

Rationale:
• Option a: A person is legally unable to sign a consent until the age of 18 years unless
the client is an emancipated minor or married, Pregnant female.
• Option b: Although the adolescent may be capable of intelligent choices, 18 is the legal
age of consent unless the client is emancipated or married.
• Option c: Parents or guardians are legally responsible under all circumstances unless
the adolescent is an emancipated minor or married.
• Option 4: Adolescents have the capacity to choose, but not the legal right in this
situation unless they are legally emancipated or married
96 National NORCET Test 6

170. A pregnant woman at 18 weeks of gestation is scheduled for an amniocentesis.


While preparing the patient for the procedure, the nurse gets to know that
the patient does not fully understand the risks and benefits associated with
the procedure. Which of the following describe the nurse’s role in obtaining
informed consent?
a. Explain the risks and benefits associated with the procedure.
b. Describe alternatives to the procedure.
c. Do nothing as such procedure does not require any consent.
d. Advocate for the client by ensuring she is making an informed decision.

Ans. d. Advocate for the client by ensuring she is making an informed decision.

Rationale:
• Option a: It is the physician’s duty to provide information to the client-related to risks
and benefits.
• Option b: It is the physician’s duty to provide information to the client related to
alternatives.
• Option c: CORRECT: One of the nurse’s roles in the informed consent process is to
witness the signature on the consent form.
• Option d: CORRECT: One of the nurse’s roles in the informed consent process is to
advocate for the client by ensuring she has been provided the necessary information to
make an informed decision.

171. Nurse Anita understands that the Inform consent is based on ……………
ethical principle?
a. Justice
b. Veracity
c. Fidelity
d. Autonomy

Ans. d. Autonomy

Rationale:
• Option d: Informed consent is based on an individual’s right to make decisions
about his or her own health care, with adequate information to make that decision.
Principle of autonomy means respect for an individual’s right to self-determination,
independence and ability to self-direct. It is an ethical action on the part of nurse to
allow the patient to make decisions
• Option a: Principle of Justice: The equitable distribution of potential benefits and tasks
determining the order in which clients should be cared and treated equally and fairly.
• Option b: Principle of veracity is defined as telling the truth. Telling the truth is basic
to communication and social relationship. Health care provider should give accurate,
reality-based information about health status and treatment prospective.
• Option c: Principle of Fidelity refers to the concept of keeping a commitment and is
based upon the virtue of caring. This principle hold that a person should be faithful
and truthful for his/her duties and obligations.
National NORCET Test 6 97

172. A nurse suspect that her patient did not give informed consent, who is posted for
laparoscopic cholecystectomy. what is her responsibility in the give situation?
a. Repeat the information to the client.
b. Contact the operative surgeon and inform.
c. Contact the family of the client, to express her concern.
d. None of the above

Ans. b. Contact the operative surgeon and inform.

Rationale:
• Option b: As a patient advocate, it is the nurse’s responsibility to ensure that the patient
has all of the necessary information and/or competence needed to give informed
consent which should be provided from the surgeon ends. So, in the given scenario the
nurse should inform to the operating surgeon about the informed consent.
• Option a: Giving information on side effects/ benefits/complication of procedure
comes under the responsibility of a surgeon.
• Option c & d: act as a distractor.

173. Which term describes a way to organize and chart patient progress?
a. SOAP b. POMR
c. PIE d. All of the above

Ans. a. SOAP

Rationale:
• Option a: SOAP Acronym
Ĕ S= Subjective
Ĕ 0= Objective
Ĕ A= Assessment
Ĕ P= Planning
SOAP method of documentation is used as organized format for making the progress notes
in the patient chart.
• Option b: POMR Acronym
P= Problem
O= oriented
M= Medical
R= Record
A method of establishing and maintaining the patient’s medical record so that problems are
clearly stated.
• Option c: PIE Acronym
P= Process
I= intervention
E= Evaluation
The progress notes in the patient record use (P) to define the particular Problem; (I) to
document Intervention; and (E) to Evaluate the patient outcome. PIE charting integrates
care planning with progress notes.
98 National NORCET Test 6

174. Archit, Registered Nurse (RN) assigned to provide care to a 56-year-old


patient diagnosed with an osteosarcoma, with right leg amputation. While
documenting the routine care, nurse accidentally documents that a dressing
changed was performed on the left leg. What would be the best action of the
nurse to correct this documentation?
a. Use correcting fluid to completely delete the error and write the correct one.
b. Draw a line just below the wrong documentation the rewrite the correct report.
c. Use Black gel pen and put cross on the statement.
d. Remove the page with the error and rewrite the data on that page correctly.

Ans. b. Draw a line just below the wrong documentation the rewrite the correct report.

Rationale:
• Option b: The nurse should not use eraser, or correcting fluids or cross mark on the
written statement in patient chart. A single line should be drawn through an incorrect
entry, and the words “mistaken entry” or “error in charting” should be printed above or
beside the entry and signed. The entry should then be rewritten correctly.
• Option a, c & d: Act as a distractors.

175. During the consultant clinical rounds, doctors need to identify the change in
health status of a postoperative patient’s vital signs. Which documents would
the nurse consult first to the doctors?
a. Admission sheet
b. Doctor’s Progress sheet
c. Graphic record
d. All of the above

Ans. c. Graphic record

Rationale:
• Option C: While one recording of vital signs should not be documented on admission
sheet, the best place to find sequential recordings that show a pattern or trend is the
graphic record (TPR Chart)
• Option a: The admission sheet does not include vital sign documentation. It contains
Demographic information of the patient and admission receipt
• Option b: Doctor’s progress sheet includes patient general condition and doctor order
related to patient health status.

176. A nurse is using the SOAP format of documentation to document care of a


patient who is diagnosed with pneumonia. Which source of information would
be the nurse’s focus when completing this documentation?
a. A patient problem list.
b. Notes describing the patient’s condition.
c. Overall trends in patient status
d. Planned interventions and patient outcomes.

Ans. a. A patient problem list.


National NORCET Test 6 99

Rationale:
• Option a: When using the SOAP format, the problem list at the front of the chart alerts
all caregivers to patient priorities.
• Option b: Narrative notes allow nurses to describe a condition, situation, or response
in their own terms.
• Option c: Abnormal status can be seen immediately when using charting by exception.
• Option d: planned interventions and patient expected outcomes are the focus of the
case management model.

177. At the end of your evening shift, one of the patients become hypoglycemic
with the blood glucose level of 45 mg/dL. You have informed to the nursing
supervisor and looking for the doctor to inform him. But unfortunately, the
doctor wasn’t available at that time. You made a call to the doctor and informed
him about the patient’s condition and received the telephonic orders. Which
of the following statement is related to the “Joint Commission read-back
requirement” policies?
a. Repeat and confirm b. Write, read back and confirm
c. Listen, repeat and confirm d. Document and proceed with the orders

Ans. b. Write, read back and confirm

Rationale:
• No verbal order by the doctors to nurses is allowed except in sterile condition, cardiac
arrest or other life-threatening situations.
• In order to reduce the errors, the “Joint Commission read-back requirement” policies
are best used for verbal or telephonic orders. The steps involve:
Ĕ First, note the date and time.
Ĕ Record the order word-for-word on the record sheet
Ĕ On the next line, write “telephone order.”
Ĕ Then write the health care provider’s name, and along with your initials
• Read back the order and get confirmation from the person who gave the order. (This
step is called the “Joint Commission read-back requirement” and applies to all verbal
and telephone orders. The read-back requirement also applies to critical test results
reported verbally or by telephone.)
• In case you are having any trouble in understanding the verbal order always ask
another nurse to take the order and she will also follow the same principle of read back
and sign the order too.
• After this as per the hospital protocol it is recommended that the doctor who has given
the verbal order has to countersign the order. Without doctor signature the nurse may
be held liable for practicing the clinical care without license.

178. A nurse is giving handover to another nurse about the patient clinical status
during her shift duty. Which action did the nurse complete?
a. Report. b. Record.
c. Consultation. d. Referral

Ans. a. Report
100 National NORCET Test 6

Rationale:
• Option a: Reports are oral, written, or audiotaped exchanges of information among
caregivers.
• Option b: A patient’s record or chart is a confidential, permanent legal document
consisting of information relevant to his or her health care.
• Option c: Consultations are another form of discussion in which one
• professional caregiver gives formal advice about the care of a patient to another
caregiver.
• Option d: referral is seeking opinion of other health department in relation to patient
condition.
• Example: Patient is admitted as a case Type 2 DM, the attending doctor order for
dietician referral.

179. Nurse Shoba performing initial assessment on her patient. After completing
the assessment, which of the following statement should the nurse document
in the patient chart?
a. Appears restless when sitting in the chair.
b. Drank adequate amounts of water.
c. Apparently is asleep with eyes closed.
d. Skin pale and cool

Ans. d. Skin pale and cool

Rationale:
• Option d: A factual record contains descriptive, objective information about what
a nurse sees, hears, feels, and smells. An objective description is the result of direct
observation and measurement. For example, “B/P 80/50, patient diaphoretic, heart rate
102 and regular.”
• Other option: Avoid vague terms such as appears, seems, or apparently because these
words suggest that you are stating an opinion, do not accurately communicate facts,
and do not inform another caregiver of details regarding behaviour’s exhibited by the
patient.
• Use of exact measurements establishes accuracy. For example, a description such as
“Intake, 360 mL of water” is more accurate than “Patient drank an adequate amount
of fluid.”

180. A clinical preceptor is working with a new nurse on documentation. Which


action made by the nurse make the preceptor to intervene?
a. The new nurse uses a black ink pen to chart.
b. The new nurse document in every other line in patient files
c. The new nurse ends each entry with signature and title.
d. The new nurse keeps the password secure

Ans. b. The new nurse document in every other line in patient files
National NORCET Test 6 101

Rationale:
• Option b: Chart consecutively, line by line (not every other line); if space is left, draw
a line horizontally through it, and sign your name at the end. Every other line should
not be left blank.
• Option a: Record all entries legibly and in black ink. The reason behind using the blank
ink it will be easily legible when we do the photocopy of the patient record and handed
over to the patient at the time of discharge.
• Option c: End each entry with your signature and title.
• Option d: If the nurse is documenting the patient data in computer-based system in
that case nurse should maintain the confidentiality and keeping the password secure
are all appropriate behaviour’s.

181. Match the correct entry with the appropriate SOAP category.
1. Repositioned patient on right side. Encouraged patient to use patient-controlled
analgesia device.
2. “the pain increases every time I try to turn on my left side”.
3. Acute pain related to tissue injury from surgical incision.
4. left lower abdominal surgical incision, 3 inches in length, closed, sutures intact, no
drainage. Pain noted on mild palpation.
a. S-2, O-3, A-4, P-1
b. S-2, O-4, A-3, P-1
c. S-3, O-2, A-4, P-1
d. S-2, O-3, A- 1, P-4

Ans. b. S-2, O-4, A-3, P-1

Rationale:
• SOAP Acronym
Ĕ S= Subjective
Ĕ 0= Objective
Ĕ A= Assessment
Ĕ P= Planning
• SOAP method of documentation is used as organized format for making the progress
notes in the patient chart.
• Other options are distractors.

NURSING EDUCATION/NURSING MANAGEMENT


182. The best method to enhance awareness of rural population toward small family
norm will be: (NE)
a. Film show
b. Chart and exhibits
c. Role playing
d. Television

Ans. c. Role playing


102 National NORCET Test 6

Rationale:
• To increase awareness of rural population toward small family norms, the best method
is role playing as in rural area most of the population is unable to read or write properly
specially the old ones. Moreover, role playing enables the group to learn by seeing and
hearing, and that places a long-lasting effect.
• Other options are incorrect as though they are also good methods of imparting health
education but for providing education on small family norm within rural population,
role play is considered the best and most effective method.

183. Which among the following principles is applied while using a teaching aid?
(NE)
a. Principle of selection
b. Principle of response
c. Principle of evaluation
d. All of the above

Ans. d. All of the above

Rationale:
• While using the teaching or audio-visual aid, all of the above principles are kept in mind.
• The Principle of selection means that the teaching aid should be selected properly
as per the objectives of the lesson plan. For instance, if a teacher wants to make her
students study about someone’s biography or history, films are considered as the best
method. Conversely, if a teacher wants to teach the diagram of neuron, then the best
teaching aid will be a “model” showing the neuron and its parts. While selecting the AV
aid teacher should be familiar with the use of it and also should see whether it develops
interest and motivation in students or not.
• Principle of response means that the AV aid acts as a stimulus to which the students
respond. This response of the students has to be guided by the teachers to ensure
proper learning.
• Principle of evaluation means that continuous evaluation of the teaching aid is
necessary to ensure that it solves the purpose efficiently for which it is used. With the
change in technology the teaching aid should also be improved and that is possible
only through regular evaluations

184. Which among the following teaching aid is an example of traditional aids? (NE)
a. Periodicals b. Info-graphs
c. Films d. Television

Ans. a. Periodicals

Rationale:
• Periodicals are the traditional aids. Traditional aids of teaching are the ones which are
based on the principles of memorization and recitation. Such techniques do not lay
emphasis on critical thinking, decision making and problem-solving skills. Some of
the examples of traditional aids are chalk-blackboard, textbooks, maps, globes, atlases,
aquarium etc.
National NORCET Test 6 103

• Other options are incorrect as they all are the teaching aids that stimulates auditory
and visual senses and makes the student think critically, solve problems and take
appropriate decisions.

185. In ABC Analysis, which of the items are frequently used and purchased in large
quantities? (NM)
a. Vital items
b. B items
c. C items
d. A items

Ans. c. C items

Rationale:
• ABC analysis system of inventory control describes about control of items where A
items are those which need tight control and accurate records.
• B items are less tightly controlled and need good records.
• C items are the simplest control, which need minimal records.
• A items are of high cost, B items of intermediate cost whereas C items are low cost
center.
• Class C or C items are correct answer which includes items those are frequently used
and purchased in large quantities such as gloves, washers, clips, etc.

186. Which set of characteristics most accurately describe Public relations? (NM)
a. Low cost, low credibility, high control
b. High cost, high credibility, low control
c. High cost, low credibility, high control
d. Low cost, high credibility, low control

Ans. d. Low cost, high credibility, low control

Rationale:
• Low cost, high credibility, low control are the characteristics that most accurately
describes the public relations (PR). Effective public relations are very important for any
organisation to flourish and for that there are some characteristics of public relation
that need to be considered, these are:
Ĕ Low cost: Public relations cost lower than the usual paid communications.
Ĕ High credibility: PR has high credibility as it develops trust in client by ensuring
the transparency of brand, providing flexible approaches, is more open to the
demands of clients and by providing customised services.
Ĕ Low control: The company has low control in terms of communication or message
content control in public relations.
Ĕ Other characteristics of public relations include:
104 National NORCET Test 6

Table: Characteristics of public relation


Characteristics of Public Relations
Public Relations
Communication Mode Usually indirect, non-personal
Communication control Moderate to low
Feedback Amount Little
Feedback Speed Delayed
Message Flow Direction One-Way
Message Content Control No
Sponsor Identification No
Reaching Large Audience Usually fast
Message Flexibility Usually no direct control
• Other options are distractors.

187. Which among the following is the type of education theory and practice that
focuses on design and use of messages which controls the learning process?
(NM)
a. AECT
b. ISPI
c. ITEA
d. ISTE

Ans. a. AECT

Rationale:
• AECT is the type of education theory and practice that focuses on design and use of
messages which controls the learning process. They used audio-visual communications
in the year 1923. At the beginning it was centred on radio. It swiftly focused on the
instructional filmstrips and educational television. Nowadays they include global
satellite broadcasting, visual communication and two-way audio.
• ISPI is another form of theory which explains the application of behaviour theory
while using the instructional system approach
• ITEA is International Technology Education Association that focuses on making
the students learn technology that is applicable in the real world like robotics,
manufacturing systems and computer-assisted designs
• ISTE lays emphasis on both the instructions and the application of computers in the
training of students
National NORCET Test 6 105

NURSING RESEARCH
188. Select which among the following is required for one to fully define nursing
research?
a. An exhaustive list of the research that has already been conducted in nursing
b. Ways to perform the major descriptive and inferential statistics needed for analysis
c. Research studies well received by other disciplines for collaborative projects
d. A determination of what nurses need to know to provide the best patient care

Ans. d. A determination of what nurses need to know to provide the best patient care

Rationale:
• Option 4: Defining nursing research requires determining the relevant knowledge
needed by nurses so that they can look and search for the new evidences which they
will incorporates into clinical practice
• Option 1: An exhaustive list is not necessary to define nursing research.
• Option 2: Nursing research is not defined by the ways to perform the major descriptive
and inferential statistics needed for analysis.
• Option 3: Nursing research is not defined by research studies well received by other
disciplines for collaborative projects.

189. A nurse is conducting quantitative research to examine the effects of following


nursing protocols in the emergency department (ED) on patient outcomes.
This is also known as what type of research?
a. Descriptive
b. Correlational
c. Quasi-experimental
d. Experimental

Ans. c. Quasi-experimental

Rationale:
• Option 3: Quasi-experimental research is often conducted in clinical settings to
examine the effects of nursing interventions on patient outcomes.
• Option1: Descriptive research is often used to generate new knowledge about topics
with little or no prior research.
• Option 2: Correlational research examines the type and degree of relationships between
two or more variables.
• Option4: Experimental research examines cause-and-effect relationships between
variables under highly controlled conditions.

190. All of the following are used in review of literature except:


a. Locating resources b. Reading notes
c. Statement of research topic d. Using library

Ans. c. Statement of research topic


106 National NORCET Test 6

Rationale:
• Statement of research topic cannot be used as review of literature as statement of
research topic is the main problem to which the researcher wants to find the solution
and for it he needs to review different literature available to him.
• Other options are incorrect because they are used as the sources of review of literature
along with magazines, newspapers, manuscripts, autobiographies, letters, eyewitness etc.

191. The following are considered steps in the qualitative research process, except?
a. Literature review b. Hypothesis
c. Sample d. Data collection

Ans. b. Hypothesis

Rationale:
• Hypothesis is not a step seen in a qualitative research process because in qualitative
research the main focus is on exploring the problem under study and not making or
testing hypothesis as done in quantitative research.
• Other options are incorrect as they all are common steps in quantitative and qualitative
research

192. What is the first step in the qualitative research process?


a. Data analysis b. Sample collection
c. Review of literature d. Study design

Ans. c. Review of literature

Rationale:
• Review of literature is the 1st step in the qualitative research process.
• Data analysis is the sixth step in the qualitative research process.
• Sampling collection is the third step in the qualitative research process.
• The study design is the second step in the qualitative research process.

193. Quasi experimental research design lack which of the main features of a true
experimental study?
a. Control b. Randomization
c. Manipulation d. Either control or randomization

Ans. d. Either control or randomization

Rationale:
• Quasi-experimental studies lack atleast one of the two main features of a true
experimental study either control or randomization and this is more often
randomisation.
• This is often because subjects in close proximity may share elements that might spoil
or ‘contaminate’ the results, or there are ethical difficulties in giving an intervention to
one group but not the other.
National NORCET Test 6 107

ENGLISH
194. Find the correctly spelt word.
a. Reconcealation b. Reconciliation
c. Reconcealiation d. Reconcilation

Ans. b. Reconciliation

Rationale:
Correctly spelt word is Reconciliation which means revival of friendly relations.

195. Tell me clearly what went wrong with this project, do not beat around the bush.
a. Giving excuses
b. Try to give justifications
c. Approach the matter in a round about way
d. Blaming others for own bad deeds

Ans. c. Approach the matter in a round about way

Rationale:
The given idiom directly states one to avoid talking about unnecessary things and state the
matter clearly.

196. Our Director is very reputed for giving splendid speeches but…
a. his today’s speech was very ambiguous.
b. his promises are always are always true.
c. is very entertaining.
d. both a & b.

Ans. a. his today’s speech was very ambiguous.

Rationale:
Only option a fits well with the given statement. Also, but is generally used to connect
contrasting ideas so option b & c gets eliminated.

197. Mark the incorrect segment of the statement: I enquired of her/ whether she
would /accompany me to my /new escapade job.
a. I enquired of her b. whether she would
c. accompany me to my d. new escapade job
e. No error

Ans. e. No error

Rationale:
There are no errors in the given sentence.
108 National NORCET Test 6

198. Arrange in a proper sequence: Without knowledge


M: cultured society can be expected
N: no growth is possible
O: and
P: without growth no
a. MOPN b. NOPM
c. OPMN d. OPNM

Ans. b. NOPM

Rationale:
Correct sentence will be:
Without knowledge no growth is possible and without growth no cultured society can be
expected.

GENERA KNOWLEDGE
199. Youth World Championship will be hosted for the first time by:
a. Hungary b. Italy
c. Serbia d. Croatia

Ans. d. Croatia

Rationale:
The International Boxing Association has given chance to Croatia for the first time to stage
the global boxing event in the year 2024.

200. Newly formed ambassador of Star Sports Channel is:


a. Ranveer Singh b. Saurabh Tiwari
c. M. S. Dhoni d. Raveesh Kumar

Ans. a. Ranveer Singh

Rationale:
Star Sports Channel is owned by Walt Disney and recently Ranveer Singh has been
announced as the brand ambassador of the channel.

201. At IBA Women’s world Boxing championship 2023, how many gold medals are
won by Indian women:
a. Four b. Three
c. Five d. No gold medal is won

Ans. a. Four
National NORCET Test 6 109

Rationale:
Four Women from India won gold medals. Nikhat Zareen, Lovlina Borgohain, Saweety
Boora and Nitu Ghanghas brought this proud moment for India.

202. India’s largest Space rocket launched on 26 March, 2023 by ISRO from Satish
Dhawan Space centre, Sriharikota carrying 36 satellites is:
a. X-ray Polarimeter Satellite (XPoSat)
b. Launch Vehicle Mark-III (LVM-III)
c. Bhaskara-I
d. SROSS-1

Ans. b. Launch Vehicle Mark-III (LVM-III)

Rationale:
Launch Vehicle Mark-III (LVM-III) deployed with 36 OneWeb satellites has been
successfully launched from Sriharikota on 26 March, 2023.

203. Recently National Genome Strategy is launched by which country?


a. India b. United States
c. Japan d. United Arab Emirates

Ans. d. United Arab Emirates

Rationale:
National Genome Strategy is launched by UAE recently in order to provide a comprehensive
framework to legislation and governance for the acceleration of the development of preventive
and personalised healthcare solutions for the citizens to improve country’s well being.

ARITHMETIC
204. The sum of ages of 5 children born at the interval of 4 years each is 70 years.
Find the age of the child born at third number.
a. 6 years b. 12 years
c. 18 years d. 14 years

Ans. d. 14 years

Rationale:
Let the age of children: x, (x+4), (x+8), (x+12), (x+16) years.
Then, x + (x+4) + (x+8) + (x+12) + (x+16) = 70
⇒ 5x = 70-40
⇒ x = 30/5
⇒ x=6
⇒ (x+8) = 6+8 = 14 years.
110 National NORCET Test 6

205. Three numbers are in the ration 6:7:8. The sum of the largest and smallest
numbers is equal to the sum of second number and 63. Find the smallest
number.
a. 39
b. 54
c. 48
d. 49

Ans. b. 54

Rationale:
A:B:C
6:7:8
Let the numbers be, 6x : 7x : 8x
Sum of smallest and largest equal to the sum of second number and 63,
A + C = B + 63
6x + 8x = 7x + 63
14x = 7x + 63
7x = 63
X=9
⇒ 6x = 6*9
⇒ 6x = 54

206. Which one of the following is completely divisible by 11?


a. 65740398
b. 27314925
c. 27314956
d. 67648987

Ans. b. 27314925

Rationale:
As per divisibility rule of 11, a number will be divisible by 11 if the difference between the
sum of digit’s odd and even numbers comes out to be either 0 or is divisible by 11.
Here, 27314925
Sum of odd numbers = 2+3+4+2=11
Sum of even numbers = 7+1+9+5=22
Difference between their sums = 22-11=11
As 11 is divisible by 11, so the given digit i.e. 27314925 is divisible by 11.
National NORCET Test 6 111

207. Ankit got an increment of 12 % this year. If his present salary is Rs. 17500, the
last year’s salary was:
a. 15625
b. 16787
c. 14879
d. 13990

Ans. a. 15625

Rationale:
Last year’s salary
= 17500/100+12 * 100
= 17500/112*100
= Rs. 15625

208. The H.C.F. of two numbers is 26 and the other two factors of their L.C.M. are 12
and 13. Find the larger of the two numbers.
a. 338
b. 297
c. 299
d. 336

Ans. a. 338

Rationale:
Numbers given are ( 26*12) and (26*13)
Larger number will be = (26*13) = 338

You might also like